Sie sind auf Seite 1von 59

1

EN BANC Vicenta de Ocampo to the questions propounded to her by the physician, appears
in the Medical Examiner's Report.
G.R. No. L-24899 March 19, 1928
On May 9, 1925, Bernardo Argente and his wife submitted to the West Coast Life
BERNARDO ARGENTE, plaintiff-appellant, Insurance Co. an amended application for insurance, increasing the amount thereof
vs. to P15,000, and asked that the policy be dated May 15, 1925. The amended
WEST COAST LIFE INSURANCE CO., defendant-appellee. application was accompanied by the documents entitled "Short Form Medical
Report." In both of these documents appear certain questions and answers.
Abad Santos, Camus, Delgado & Recto for appellant.
Gibbs & McDonough and Roman Ozaeta for appellee. A temporary policy for P15,000 was issued to Bernardo Argente and his wife as of
May 15, but it was not delivered to Bernardo Argente until July 2, 1925, when the
MALCOLM, J.: first quarterly premium on the policy was paid. In view of the fact that more than
thirty days had elapsed since the applicants were examined by the company's
physician, each of them was required to file a certificate of health before the policy
This is an action upon a joint life insurance policy for P15,000 issued by the
was delivered to them.
defendant, the West Coast Life Insurance Co., on May 15, 1925, in favor of the
plaintiff, Bernardo Argente, and his wife, Vicenta de Ocampo, the latter having died
on November 18, 1925. Fraud in obtaining the policy was pleaded by way of special On November 18, 1925, Vicenta de Ocampo died of cerebral apoplexy. Thereafter
defense. On the issue thus suggested, the court adopted the theory of the Bernardo Argente presented a claim in due form to the West Coast Life Insurance
defendant, and held the insurance policy null and void, with the result that the Co. for the payment of the sum of P15,000 the amount of the joint life Insurance
complaint was dismissed, with costs. policy. Following investigation conducted by the Manager of the Manila office of
the insurance company, it was apparently disclosed that the answers given by the
insured in their medical examinations with regard to their health and previous
On February 9, 1925, Bernardo Argente signed an application for joint insurance
illness and medical attendance were untrue. For that reason, the West Coast Life
with his wife in the sum of P2,000. The wife, Vicenta de Ocampo, signed a like
Insurance Co. refused to pay the claim of Bernardo Argente, and on May 25, 1926,
application for the same policy. Both applications, with the exception of the names
wrote him to the effect that the claim was rejected because the insurance was
and the signatures of the applicants, were written by Jose Geronimo del Rosario, an
obtained through fraud and misrepresentation.
agent for the West Coast Life Insurance Co. But all the information contained in the
applications was furnished the agent by Bernardo Argente.
It is admitted that it appears in the Medical Examiner's Report that Bernardo
Argente, in response to the question asked by the medical examiner, "Have you
Pursuant to his application, Bernardo Argente was examined by Dr. Cesareo Sta.
ever consulted a physician for, or have you ever suffered from any ailment or
Ana, a medical examiner for the West Coast Life Insurance Co., on February 10,
disease of, the brain or nervous system?" answered "No." To the question, "Have
1925, in the office of the Customs House. The result of such examination was
you consulted a physician for any ailment or disease not included in your above
recorded in the Medical Examiner's Report, and with the exception of the signature
answer," answered "Yes. Nature of Ailment, Disease or Injury. Scabies, Number of
of Bernardo Argente, was in the hand-writing of Doctor Sta. Ana. But the
attacks 1, Date 1911. Duration 1 month, Severity Fair, results and, if within five
information or answers to the questions contained on the face of the Medical
years, name and address of every physician consulted. Dr. P. Guazon. Cured. Dr.
Examiner's Report were furnished the doctor by the applicant, Bernardo Argente.
Guazon is dead now." And to the question, "What physician or physicians, if any,
not named above, have you consulted or been treated by, within the last five years
Pursuant to her application, Vicenta de Ocampo, wife of the plaintiff, was examined
and for what illness or ailment? (If none, so state)" answered "No." It is, however,
by Dr. Cesareo Sta. Ana on February 10, 1925, at her residence in Manila. The result
not disputed that on January 10, 11, and 13, 1923, Bernardo Argente was confined
of the medical examination, including among other things, the answers given by
in the Philippine General Hospital where he was treated by Dr. Agerico B. M. Sison
for cerebral congestion and Bell's Palsy.
2

It is further admitted that it appears in the Medical Examiner's Report that Vicenta here be accepted since the stenographic transcript is incomplete, the question
de Ocampo, in response to the question asked by the medical examiner, "How arises as to the estate of the law in relation thereto.
frequently, if at all, and in what quantity do you use beer, wine, spirits or other
intoxicants?" answered "Beer only in small quantities occasionally." To the One ground for the rescission of a contract of insurance under the Insurance Act is
question, "Have you ever consulted a physician for or have you ever suffered from "a concealment," which in section 25 is defined as "A neglect to communicate that
any ailment or disease of the brain or nervous system?" answered "No." To the which a party knows and ought to communicate." Appellant argues that the alleged
question, "What physician or physicians, if any, not named above, have you concealment was immaterial and insufficient to avoid the policy. We cannot agree.
consulted or been treated by, within the last five years and for what illness or In an action on a life insurance policy where the evidence conclusively shows that
ailment? (If none, so state)" answered "None." And to the question, "Are you in the answers to questions concerning diseases were untrue, the truth of falsity of
good health as far as you know and believe?" answered "Yes." It is, however, not the answers become the determining factor. In the policy was procured by
disputed that Vicenta de Ocampo was taken by a patrolman, at the request of her fraudulent representations, the contract of insurance apparently set forth therein
husband, Bernardo Argente, on May 19, 1924, to the Meisic police station, and was never legally existent. It can fairly be assumed that had the true facts been
from there was transferred to the San Lazaro Hospital. In San Lazaro Hospital, her disclosed by the assured, the insurance would never have been granted.
case was diagnosed by the admitting physician as "alcoholism," but later Doctor
Domingo made a diagnosis of probable "manic-depressive psychosis," and still, later In Joyce, The Law of Insurance, second edition, volume 3, Chapter LV, is found the
in Mary Chiles Hospital, made a final diagnosis of "phycho-neurosis." following:

The plaintiff, Bernardo Argente, while readily conceding most of the facts herein Concealment exists where the assured has knowledge of a fact material to
narrated, yet alleges that both he and his wife revealed to the company's physician. the risk, and honesty, good faith, and fair dealing requires that he should
Doctor Sta. Ana, all the facts concerning the previous illnesses and medical communicate it to the assured, but he designated and intentionally with
attendance, but that Doctor Sta. Ana, presumably acting in collusion, with the holds the same.
insurance agent, Jose Geronimo del Rosario, failed to record them in the medical
reports. The evidence on these points consists of the testimony of the plaintiff and
Another rule is that if the assured undertakes to state all the circumstances
his subordinate clerk, Apolonio Espiritu, on the one hand, and of the testimony of
affecting the risk, a full and fair statement of all is required.
Doctor Sta. Ana and Jose Geronimo del Rosario on the other. On the question of
fact thus raised, the trial judge found with the insurance company. In so doing, we
It is also held that the concealment must, in the absence of inquiries, be
believe that His Honor gave proper inclination to the weight of the proof. There
not only material, but fraudulent, or the fact must have been intentionally
appears no motive whatever on the part of Doctor Sta. Ana to falsify the Medical
withheld; so it is held under English law that if no inquiries are made and
Examiner's Reports and thereby not only jeopardize his career as a physician, but
no fraud or design to conceal enters into the concealment the contract is
also gravely implicate himself criminally.
not avoided. And it is determined that even though silence may constitute
misrepresentation or concealment it is not itself necessarily so as it is a
What has heretofore been stated in this decision is gleaned to a great extent the
question of fact. Nor is there a concealment justifying a forfeiture where
carefully prepared decision of the trial judge, the Honorable George R. Harvey. The
the fact of insanity is not disclosed no questions being asked concerning
court found from the evidence that the representations made by Bernardo Argente
the same. . . .
and his wife in their applications to the defendant for life insurance were false with
respect to their estate of health during the period of five years preceding the date
But it would seem that if a material fact is actually known to the assured,
of such applications, and that they knew the representations made by them in their
its concealment must of itself necessarily be a fraud, and if the fact is one
applications were false. The court further found from the evidence that the answers
which the assured ought to know, or is presumed to know, the
given by Bernardo Argente and his wife at the time of the medical examination by
presumption of knowledge ought to place the assured in the same position
Doctor Sta. Ana were false with respect to the condition of their health at that time
as in the former case with relation to material facts; and if the jury in such
and for a period of several years prior thereto. Based on these findings which must
cases find the fact material, and one tending to increase the risk, it is
3

difficult to see how the inference of a fraudulent intent or intentional to material facts the policy is avoided without regard to the knowledge or
concealment can be avoided. And it is declared that if a material fact fraud of assured, although under the statute statements are
concealed by assured it is equivalent to a false representation that it does representations which must be fraudulent to avoid the policy. So under
not exist and that the essentials are the truth of the representations certain codes the important inquiries are whether the concealment was
whether they were intended to mislead and did insurer accept them as willful and related to a matter material to the risk.
true and act upon them to his prejudice. So it is decided that under a
stipulation voiding the policy for concealment or misrepresentation of any If the assured has exclusive knowledge of material facts, he should fully
material fact or if his interest is not truly stated or is either than the sole and fairly disclose the same, whether he believes them material or not. But
and unconditional ownership the facts are unimportant that insured did notwithstanding this general rule it will not infrequently happen, especially
not intend to deceive or withhold information as to encumbrances even in life risks, that the assured may have a knowledge actual or presumed of
though no questions were asked. And if insured while being examined for material facts, and yet entertain an honest belief that they are not
life insurance and knowing that she had heart disease, falsely stated that material. . . . The determination of the point whether there has or has not
she was in good health, and though she could not read the application, it been a material concealment must rest largely in all cases upon the form
was explained to her and the questions asked through an interpreter, and of the questions propounded and the exact terms of the contract. Thus,
the application like the policy contained and provision that no liability where in addition to specifically named diseases the insured was asked
should be incurred unless the policy was delivered while the insured was in whether he had any sickness within ten years, to which he answered "No,"
good health, the court properly directed a verdict for the insurer, though a and it was proven that within that period he had a slight of pharyngitis, it
witness who was present at the examination testified that the insured was was held a question properly for the jury whether such an inflammation of
not asked whether she had heart disease. the throat was a "sickness" within the intent of the inquiry, and the court
remarked on the appeal decision that if it could be held as a matter of law
The basis of the rule vitiating the contract in case of concealment is that it that the policy was thereby avoided, then it was a mere device on the part
misleads or deceives the insurer into accepting the risk, or accepting it at of insurance companies to obtain money without rendering themselves
the rate of premium agreed upon. The insurer, relying upon the belief that liable under the policy. . . .
the assured will disclose every material within his actual or presumed
knowledge, is misled into a belief that the circumstance withheld does not . . . The question should be left to the jury whether the assured truly
exist, and he is thereby induced to estimate the risk upon a false basis that represented the state of his health so as not mislead or deceive the
it does not exist. The principal question, therefore, must be, Was the insurer; and if he did not deal a good faith with insurer in that matter, that
assurer misled or deceived into entering a contract obligation or in fixing the inquiry should be made, Did he know the state of his health so as to be
the premium of insurance by a withholding of material information of facts able to furnish a proper answer to such questions as are propounded? A
within the assured's knowledge or presumed knowledge? Massachusetts case, if construed as it is frequently cited, would be
opposed to the above conclusion; but, on the contrary, it sustains it, for
It therefore follows that the assurer in assuming a risk is entitled to know the reason that symptoms of consumption had so far developed
every material fact of which the assured has exclusive or peculiar themselves within a few months prior to effecting the insurance as to
knowledge, as well as all material facts which directly tend to increase the induce a reasonable belief that the applicant had that fatal disease, and we
hazard or risk which are known by the assured, or which ought to be or are should further construe this case as establishing the rule that such a
presumed to be known by him. And a concealment of such facts vitiates matter cannot rest alone upon the assured's belief irrespective of what is a
the policy. "It does not seem to be necessary . . . that the . . . suppression reasonable belief, but that it ought to be judged by the criterion whether
of the truth should have been willful." If it were but an inadvertent the belief is one fairly warranted by the circumstances. A case in Indiana,
omission, yet if it were material to the risk and such as the plaintiff should however, holds that if the assured has some affection or ailment of one or
have known to be so, it would render the policy void. But it is held that if more of the organs inquired about so well-defined and marked as to
untrue or false answers are given in response to inquiries and they relate materially derange for a time the functions of such organ, as in the case of
4

Bright's disease, the policy will be avoided by a nondisclosure, irrespective


of the fact whether the assured knew of such ailment or not. . . .

Lastly, appellant contends that even if the insurance company had a right to rescind
the contract, such right cannot now be enforced in view of the provisions of section
47 of the Insurance Act providing "Whenever a right to rescind a contract of
insurance is given to their insurer by provision of this chapter, such right must be
exercised previous to the commencement of an action on the contract." This
section was derived from section 2583 of the California Civil Code, but in contrast
thereto, makes use of the imperative "must" instead of the permissive "may."
Nevertheless, there are two answers to the problem as propounded. The first is
that the California law as construed by the code examiners, at whose
recommendation it was adopted, conceded that "A failure to exercise the right (of
rescission), cannot, of course, prejudice any defense to the action which the
concealment may furnish." (Codes of California annotated; Tan Chay Heng vs. West
Coast Life Insurance Company [1927], p. 80,ante.) The second answer is that the
insurance company more than one month previous to the commencement of the
present action wrote the plaintiff and informed him that the insurance contract was
void because it had been procured through fraudulent representations, and offered
to refund to the plaintiff the premium which the latter had paid upon the return of
the policy for cancellation. As held in California as to a fire insurance policy, where
any of the material representations are false, the insurer's tender of the premium
and notice that the policy is canceled, before the commencement of suit thereon,
operate to rescind the contract of insurance. (Rankin vs.Amazon Insurance Co.
[1891], 89 Cal., 203.)

We are content to rest our judgment on the findings of the trial court, and on the
law governing those facts, with the result that the various assignments of error are
found to be without persuasive merit.

Judgment affirmed, with the costs of this instance against the appellant.
5

Argente v. West Coast Life Insurance Co.- Misrepresentation questions concerning diseases were untrue, the truth or falsity of the answer
becomes the determining factor.
51 PHIL 725
If the policy was procured by fraudulent misrepresentations, the contract of
Facts: insurance apparently set forth therein was never legally existent. It can be fairly
assumed that had the true facts been disclosed by the insured, the insurance would
> A joint life insurance policy was issued to Bernardo Argente and his wife Vicenta
never have been granted.
upon payment of premium, by West Coast.

> On Nov. 18, 1925, during the effectivity of the policy, Vicenta died of cerebral
apoplexy. Thereafter, Bernardo claimed payment but was refused. Argente v West Coast G.R. No. L-24899 March 19, 1928
J. Malcolm
> It is admitted that in the Medical Examiner’s report, Vicenta, in response to the
question asked by the medical examiner, her replies were as follows: Facts:
o “How frequently do you use beer, wine, spirits and other intoxicants?” she Bernardo Argente signed an application for joint insurance with his wife in the sum
answered “beer only in small quantities”. of P2,000. The wife, Vicenta de Ocampo, signed for the same. All the information
contained in the applications was furnished the agent by Bernardo Argente.
o “What physician have you consulted or been treated by within the last 5 years Argente was examined by Dr. Sta. Ana, a medical examiner for the West Coast. The
and for what illness or ailment?” she answered “none” result was recorded in the Medical Examiner's Report, and with the exception of
> It is however, not disputed that in 1924, Vicenta was taken to a hospital for what the signature of Bernardo Argente, was in the hand-writing of Doctor Sta. Ana. But
the information or answers to the questions contained on the face of the Medical
was first diagnosed as alcoholism and later changed to manic-depressive psychosis
Examiner's Report were furnished the doctor by Argente.
and then again changed to pscyhonuerosis.
Vicenta de Ocampo, wife of the plaintiff, was examined at her residence by the
Issue: same doctor.
The spouses submitted to West Coast Life an amended application, increasing the
Whether or not on the basis of the misrepresentations of Vicenta, Bernardo is amount to P15,000, and asked that the policy be dated May 15, 1925. The amended
barred from recovery. application was accompanied by the documents entitled "Short Form Medical
Report." In both of these documents appear certain questions and answers.
Held:
A temporary policy for P15,000 was issued to Bernardo Argente and his wife as of
YES. May 15, but it was not delivered until the first quarterly premium on the policy was
paid. More than thirty days had elapsed since the applicants were examined. Each
The court found that the representations made by Vicenta in his application for life of them was required to file a certificate of health before the policy was delivered.
insurance were false with respect to her state of health and that she knew and was Vicenta de Ocampo died of cerebral apoplexy. Argente presented a claim in due
aware that the representations so made by her were false. In an action on a life form to the West Coast Life Insurance Co. for the payment of the sum of P15,000. It
insurance policy where the evidence conclusively shows that the answers to was apparently disclosed that the answers given by the insured in their medical
6

examinations with regard to their health were untrue. West Coastrefused to pay are presumed to be known by him. And a concealment of such facts vitiates the
the claim and wrote Argente to the effect that the claim was rejected due to fraud. policy.
The trial court held the policy null and void, hence this appeal. If the assured has exclusive knowledge of material facts, he should fully and fairly
disclose the same, whether he believes them material or not. The determination of
Issue: WON Argente and Ocampo were guilty of concealment and thereby misled the point whether there has or has not been a material concealment must rest
the insurer into accepting the risk? largely in all cases upon the exact terms of the contract.

Held: Yes. Petition dismissed.


51 Phil. 725 – Mercantile Law – Insurance Law – Representation – Concealment –
Ratio: Rescission of an Insurance Contract
Vicenta de Ocampo, in response to the question asked by the medical examiner,
In February 1925, Bernardo Argente and his wife applied for a joint life insurance
answered no to "Have you ever consulted a physician for or have you ever suffered
under West Coast Life Insurance Company (West Coast). The couple was examined
from any ailment or disease of the brain or nervous system?" She also answered
“none” as to the question whether she consumed alcohol of not. by the insurance company doctor (Doctor Sta. Ana). The couple disclosed to the
To the question, "What physician or physicians, if any, not named above, have you doctor that they never had any serious medical histories; that they were never
consulted or been treated by, within the last five years and for what illness or confined; that Vicenta De Ocampo (wife of Argente) was not an alcoholic. Doctor
ailment?" she answered "None." Sta. Ana then recommended the approval of the application. In May 1925, the
But the facts show that she was taken to San Lazaro Hospital, her case was couple were issued with the insurance policy. In November 1925, Vicenta died.
diagnosed by the admitting physician as "alcoholism”, moreover, she was diagnosed West Coast Life denied the subsequent insurance claim filed by Argente as it
with "phycho-neurosis."
averred that the application made in June was attended by fraud because the
Section 25 of the Insurance Code defined concealment as "a neglect to
couple failed to disclose the fact that each of them were actually confined prior to
communicate that which a party knows and ought to communicate."
The court held that the alleged concealment was not immaterial and insufficient to their application; that Vicenta in particular was diagnosed for alcoholism and
avoid the policy. In an action on a life insurance policy where the evidence ultimately for psycho-neurosis; that in sum, their statement as to their health and
conclusively shows that the answers to questions concerning diseases were untrue, previous illnesses within the last 5-7 years prior to their application were untrue.
the truth of falsity of the answers become the determining factor. If the true facts
Argente conceded to the allegations of West Coast however he stated that those
been disclosed by the assured, the insurance would never have been granted.
facts were actually disclosed to Dr. Sta. Ana however Dr. Sta. Ana connived with the
Concealment must, in the absence of inquiries, be not only material, but fraudulent,
or the fact must have been intentionally withheld. If no inquiries are made and no insurance agent hence he failed to record them in the medical reports. Further,
fraud or design to conceal enters into the concealment the contract is not avoided. Argente averred that if West Coast did have the right to rescind the insurance, it
The assurer is entitled to know every material fact of which the assured has should have done so prior to the filing of a suit involving the insurance claim.
exclusive or peculiar knowledge, as well as all material facts which directly tend to
ISSUE: Whether or not Argente is entitled to the insurance claim.
increase the hazard or risk which are known by the assured, or which ought to be or
7

HELD: No. In an action on a life insurance policy where the evidence conclusively
shows that the answers to questions concerning diseases were untrue, the truth or
falsity of the answers become the determining factor. If the policy was procured by
fraudulent representations, the contract of insurance apparently set forth therein
was never legally existent. It can fairly be assumed that had the true facts been
disclosed by the assured, the insurance would never have been granted. The
allegations of Argente do not have a leg to stand on, Dr. Sta. Ana has no motive
whatsoever and such alleged illicit act will only destroy his reputation as a
physician.

As to the allegation of Argente regarding the failure of West Coast to rescind the
insurance prior to the filing of this case, there are two answers:

1. The failure of West Coast to rescind the contract cannot prejudice any defense to
the suit which concealment may furnish.

2. Prior to the filing of this case, West Coast sent a notice to Argente advising him
that the policy is being canceled due to the concealment and that his premium is
being refunded – this operates as a rescission to the contract of insurance.
8

EN BANC at the legal rate, the sum of P10,000.00 as moral damages, the further sum of
P3,000.00 as attorney’s fees, and the costs of action.
[G.R. No. L-12465. May 29, 1959.]
Defendant, in its answer, set up the defense that the insured was guilty of
misrepresentation and concealment of material facts in that he gave false and
YU PANG CHENG alias YU PANG CHING, Petitioner, v. THE COURT OF APPEALS, ET untruthful answers to certain questions asked him in his application for insurance
AL.,Respondents. which were material to the risk insured against and have the effect of avoiding the
insurance policy.
M. de la Rosa and Yuseco, Abdon, Yuseco & Narvasa for Petitioner.
After trial, the court rendered judgment ordering defendant to pay plaintiff the sum
Perkin & Ponce Enrile for Respondents. of P10,000.00, with legal interest from the filing of the complaint, plus the sum of
P2,000.00 as attorney’s fees, and the costs of suit. On appeal, the Court of Appeals
reversed the decision of the trial court, holding that the insured was guilty from
liability. Hence the present petition for review.
SYLLABUS
On September 5, 1950, Yu Pang Eng submitted parts II and III of his application for
insurance consisting if the medical declaration made by him to the medical
1. INSURANCE; WORDS AND PHRASES; "CONCEALMENT." — "A neglect to examiner of defendant and the medical examiner’s report. On September 7, he
communicate that which a party knows and ought to communicate is called submitted part I of his application which is the declaration made by him to an agent
concealment." (Section 25, Act NO. 2427.) of defendant, and on September 8, based on said application, and upon payment of
the first premium in the sum of P591.70, defendant issued to the insured entered
2. ID.; CONCEALMENT AS GROUND FOR RESCESSION OF CONTRACT. — Whenever Policy No. 812858.
intentional the concealment entitles the insurer to rescind the contract of
insurance. (Section 26 of Act No. 2427.) On December 27, 1950, the insured entered St. Luke’s Hospital for medical
treatment but he died on February 27, 1951. According to the death certificate, he
3. ID.; DUTY OF INSURED TO COMMUNICATE ALL FACTS TO INSURER. — The died of "infitrating medullary carcinoma, Grade 4, advanced catdiac and of lesser
insurance law requires the insured to communicate to the insurere all facts within curvature, stomach metastases spleen." Plaintiff, brother and beneficiary of the
his knowledge which are material to the contract and which the other party has not insured, demanded from defendant the payment of the proceeds of the insurance
the means of ascertaining (Section 27), and the materiality is to be determined not policy and when the demand was refused, he brought the present action.
by the event but solely by the probable and reasonable influence of the facts upon
the party to whom the communication is due (Section 30 of Act 2427.) The issue to be determined is whether the insured is guilty of concealment of some
facts material to the risk insured against which has the effect of avoiding the policy
as found by respondent court.
DECISION
The insured, in his application for insurance, particularly in his declarations to the
examining physician, stated the following in answerubg the questions propounded
BAUTISTA ANGELO, J.: to him:jgc:chanrobles.com.ph

"14. Have you ever had any of the following diseases or symptoms? Each question
Plaintiff brought this action to collect from defendant the sum of P10,000.00, value must be read and answered "Yes" or "No."
of an insurance policy taken upon the life of one Yu Pand Eng, plus interest thereon
"Gastritis, Ulcer of the Stomach or any disease of that organ? No.
9

2427). Whether intentional or unintentional, the concealment entitles the insurer


"Vertigo, Dizziness, Fainting-spells or Unconsciousness? No. to rescind the contract of insurance (Section 26). Our law even requires the insured
to communicate to the insurer all facts within his knowledge which are material to
"Cancer, Tumors or Ulcers of any kind? No. the contract and which the other partty has not the means of ascertaining (Section
27), and the materiality is to be determined not by the event but solely by the
"15. Have you ever consulted any physician not included in any of the above probable and reasonable influence of the facts upon the party to whom the
answers? Give names and address or physicians list ailments or accidents and date. communication is due (Section 30).
No."cralaw virtua1aw library
In the case of Argente v. West Coast Life Insurance Co., 51 Phil., 725, this Court
It appears that the insured entered the Chinese General Hospital for medical said:jgc:chanrobles.com.ph
treatment on January 29, 1950having stayed there up to February 11, 1950. Upon
entering the hospital, he complained of dizziness, anemia, abdominal paids and "One ground for the rescission of a contract of insurance under the Insurance Act is
tarry stools, and in the evening of his admission he had several abdominal pains and ‘a concealment’, which in section 25 is defined ‘A neglect to communicate that
his discharges were with black tarry stools and felt dizzy and weak. The history of which a party knows and ought to communicate.’ Appellant argues that the
his illness shows that the same "started a year ago as frequent dizziness." An X-ray concealment was immaterial and isufficeint to avoid the policy. We cannot agree. In
picture of his stomach was taken and the diagnosis made of him by his doctors an action on a life insurance policy where the evidence conclusively shows that the
showed that his illness was "peptic ulcer, bleeding."cralaw virtua1aw library answers to questions concerning diseases were untrue, the truth or falsity of the
answers become the determining factor. If the policy was procured by fraudulent
It should be noted that the insured’s confinementt in the Chinese General Hospital representations, the contract of insurance apparently set forth therein was never
took place from January 29, 1950 to February 11, 1950, whereas his application for legally existent. It can fairly be assumed become that had the true facts been
insurance wherein he stated his answers to the questions propounded to him by disclosed by he assured, the insurance would never have been granted."cralaw
the examining physician of defendant was submitted to defendant on September 5, virtua1aw library
1950. It is apparent that when the insured gave his answers regarding his previous
ailment, particularly with regard to "Gaztritis, Ulcer of the Stomach or any disease Upon the foregoing reasons, we are persuaded to conclude that respondent court
of that organ" and "Vertigo, Dizziness, Fainting-spells or Unconsciousness", he did not err in declaring the policy ineffective on the ground of concealment and in
concealed the ailment of which he was treated in the Chinese General Hospital relieving appellee from liability thereunder.
which precisely has direct connection with the subject of the questions
propounded. The negative answers given by the insured regarding his previous Wherefore, the decision appealed from is affirmed, with costs against Petitioner-
ailment, or his concealment of the fact that he was hospitalized and treated for Appellant.
sometime of peptic ulcer and had suffered form "dizziness, anemia, abdominal
pains and tarry stools", deprived defendant of the opportunity to make the
necessary inquiry as to the nature of his past illness so that it may form its estimate
relative to the approval of his application. Had defendant been given such
opportunity, considering the previous illness of the insured as disclosed by the
records of the Chinese General Hospital, defendant would probably had never
consented to the issuance of the policy in question. In fact, according to the death
certificate, the insured died of "infiltrating medullary carcinoma, Grade 4, advanced
cardiac and of lesser curvature, stomach metastases spleen", which may have a
direct connection with his previous illness.

Our Insurance Law provides that A neglect to communicate that which a party
knows and ought to communicate, is called concealment" (Section 25, Act No.
10

Facts: according to the death certificate, the insured’s death may have direct connection
with his previous illness.
Yu Pang Eng submitted application for insurance consisting of the medical Under the law, a neglect to communicate that which a party knows and ought to
declaration made by him to the medical examiner and the report. Yu then paid the communicate, is called concealment. This entitles the insurer to rescind the
premium in the sum of P591.70. contract. The insured is required to communicate to the insurer all facts within his
The insured, in his application for insurance, said “no” to ever having stomach knowledge which are material to the contract and which the other party has not the
disease, cancer, and fainting-spells. He also claimed to not have consulted a means of ascertaining. The materiality is to be determined not by the event but
physician regarding such diseases. solely by the probable and reasonable influence of the facts upon the party to
After submitting the form, he entered the hospital where he complained of whom the communication is due.
dizziness, anemia, abdominal pains and tarry stools. He was found to have peptic Argente vs. West Coast- “One ground for the rescission of a contract of insurance
ulcer. under the insurance Act is "a concealment", which in section 25 is defined "A
The insured entered another hospital for medical treatment but he died of neglect to communicate that which a party knows and ought to communicate."
"infiltrating medullary carcinoma, Grade 4, advanced cardiac and of lesser “In an action on a life insurance policy where the evidence conclusively shows that
curvature, stomach metastases spleen." the answers to questions concerning diseases were untrue, the truth or falsity of
Yu Pang Cheng aimed to collect P10,000.00 on life of one Yu Pang Eng from an the answers become the determining factor. If the policy was procured by
insurance company. fraudulent representations, the contract of insurance was never legally existent. It
The company set up the defense that the insured was guilty of misrepresentation can fairly be assumed that had the true facts been disclosed by the assured, the
and concealment of material facts. They subsequently refused to give the insurance would never have been granted.”
indemnity.
The trial court rendered judgment ordering defendant to pay plaintiff the sum of
P10,000.00, plus P2,000.00 as attorney's fees. The Court of Appeals reversed the Yu Pang Cheng v. CA- Life Insurance Policy
decision of the trial court, holding that the insured was guilty of concealment of
material facts. Hence the present petition. 105 PHIL 1930

Issue: Whether or not the insured is guilty of concealment of some facts material to Facts:
the risk insured that consequently avoids the policy.
> Yu Pang Eng obtained a life insurance policy naming his brother Yu Pang Cheng as
Held: Yes. Petition dismissed. beneficiary.

> Eng subsequently died of medullary carcinoma, Grade 4, advanced and lesser
Ratio: curvature.
The first confinement took place from January 29, 1950 to February 11, while his
application was submitted on September 5, 1950. When he gave his answers to the > Cheng claims the proceeds of the policy.
policy, he concealed the ailment of which he was treated in the hospital. > Insurance co. refused payment on the ground that the policy was void due to the
The negative answers given by the insured regarding his previous ailment deprived concealment.
defendant of the opportunity to make the necessary inquiry as to the nature of his
past illness so that as it may form its estimate relative to the approval of his Issue:
application. Had defendant been given such opportunity, the company would
probably had never consented to the issuance of the policy in question. In fact, Whether or not the policy is void.
11

Held: event but by the probable and reasonable influence of the facts upon the party to
whom the communication is due.
YES. The insured’s negative answers to the questions on his previous ailments,
In the application for the policy, Eng was asked whether he had been ill or had or his concealment of his hospitalization deprived the insurance company of the
opportunity to make the necessary inquiry as to the nature of his past illness so that
consulted a doctor due to symptoms or illnesses enumerated in the questionnaire.
it may form its estimate relative to the approval of his application. Had the
He answered “ No”, when in fact he was hospitalized seven months prior to his
insurance company been given such opportunity, it would not probably consent to
application for the said policy. the policy issuance.

Yu Pang Cheng v. CA

YU PANG CHENG v. CA
1959 / Bautista Angelo / Petition for review by certiorari of a CA decision
FACTS
On September 1950, Yu Pang Eng submitted his application for insurance to an
insurance company [defendant]. He answered “no” to questions on his medical
history (stomach diseases, dizziness,ulcers, vertigo, cancer, tumors, etc.) as well as
to the question of WON he consulted any physician regarding said diseases. Upon
payment of the first premium, the company issued to him an insurance policy. On
December 1950, he went to St. Luke’s for medical treatment but he died two
months later. According to the death certificate, he died of infiltrating medullary
carcinoma, Grade 4, advanced cardiac and of lesser curvature, stomach metastases
spleen.
His brother and beneficiary, Yu Pang Cheng [petitioner], demanded from
the insurance company the payment of the policy proceeds [10k], but his demand
was refused so he brought the present action. The insurance company’s defense
was that the insured was guilty of misrepresentation and concealment of material
facts in that he gave false and untruthful answers to questions asked him in his
application; hence, the effect is the avoiding of the policy.
It appears that the insured entered the Chinese General Hospital for
medical treatment on January 1950 [before application for insurance policy],
complaining of dizziness, anemia, abdominal pains and tarry stools. His illness
history shows that this started a year ago as frequent dizziness. An x-ray picture of
his stomach and the diagnosis was that he suffered from peptic ulcer, bleeding.
INSURED IS GUILTY OF CONCEALMENT OF MATERIAL FACTS
Concealment is a neglect to communicate that which a party knows and ought to
communicate. Whether intentional or not, concealment entitles the insurer to
rescind the contract. The law requires the insured to communicate to the insurer all
facts within his knowledge which are material to the contract and which the other
party has not the means of ascertaining. The materiality is determined not by the
12

EN BANC discharged, although according to the surgeon who operated on her she could not
be considered definitely cured, her ailment being of the malignant type.
G.R. No. L-16163 February 28, 1963
Notwithstanding the fact of her operation Estefania A. Saturnino did not make a
IGNACIO SATURNINO, in his own behalf and as the JUDICIAL GUARDIAN OF disclosure thereof in her application for insurance. On the contrary, she stated
CARLOS SATURNINO, minor, plaintiffs-appellants, therein that she did not have, nor had she ever had, among other ailments listed in
vs. the application, cancer or other tumors; that she had not consulted any physician,
THE PHILIPPINE AMERICAN LIFE INSURANCE COMPANY, defendant-appellee. undergone any operation or suffered any injury within the preceding five years; and
that she had never been treated for nor did she ever have any illness or disease
Eleazaro A. Samson for plaintiffs-appellants. peculiar to her sex, particularly of the breast, ovaries, uterus, and menstrual
Abello & Macias for defendant-appellee. disorders. The application also recites that the foregoing declarations constituted "a
further basis for the issuance of the policy."
MAKALINTAL, J.:
The question at issue is whether or not the insured made such false representations
of material facts as to avoid the policy. There can be no dispute that the
Plaintiffs, now appellants, filed this action in the Court of First Instance of Manila to
information given by her in her application for insurance was false, namely, that she
recover the sum of P5,000.00, corresponding to the face value of an insurance
had never had cancer or tumors, or consulted any physician or undergone any
policy issued by defendant on the life of Estefania A. Saturnino, and the sum of
operation within the preceding period of five years. Are the facts then falsely
P1,500.00 as attorney's fees. Defendant, now appellee, set up special defenses in its
represented material? The Insurance Law (Section 30) provides that "materiality is
answer, with a counterclaim for damages allegedly sustained as a result of the
to be determined not by the event, but solely by the probable and reasonable
unwarranted presentation of this case. Both the complaint and the counterclaim
influence of the facts upon the party to whom the communication is due, in forming
were dismissed by the trial court; but appellants were declared entitled to the
his estimate of the proposed contract, or in making his inquiries." It seems to be the
return of the premium already paid; plus interest at 6% up to January 8, 1959, when
contention of appellants that the facts subject of the representation were not
a check for the corresponding amount — P359.65 — was sent to them by appellee.
material in view of the "non-medical" nature of the insurance applied for, which
does away with the usual requirement of medical examination before the policy is
The policy sued upon is one for 20-year endowment non-medical insurance. This
issued. The contention is without merit. If anything, the waiver of medical
kind of policy dispenses with the medical examination of the applicant usually
examination renders even more material the information required of the applicant
required in ordinary life policies. However, detailed information is called for in the
concerning previous condition of health and diseases suffered, for such information
application concerning the applicant's health and medical history. The written
necessarily constitutes an important factor which the insurer takes into
application in this case was submitted by Saturnino to appellee on November 16,
consideration in deciding whether to issue the policy or not. It is logical to assume
1957, witnessed by appellee's agent Edward A. Santos. The policy was issued on the
that if appellee had been properly apprised of the insured's medical history she
same day, upon payment of the first year's premium of P339.25. On September 19,
would at least have been made to undergo medical examination in order to
1958 Saturnino died of pneumonia, secondary to influenza. Appellants here, who
determine her insurability.
are her surviving husband and minor child, respectively, demanded payment of the
face value of the policy. The claim was rejected and this suit was subsequently
Appellants argue that due information concerning the insured's previous illness and
instituted.
operation had been given to appellees agent Edward A. Santos, who filled the
application form after it was signed in blank by Estefania A. Saturnino. This was
It appears that two months prior to the issuance of the policy or on September 9,
denied by Santos in his testimony, and the trial court found such testimony to be
1957, Saturnino was operated on for cancer, involving complete removal of the
true. This is a finding of fact which is binding upon us, this appeal having been taken
right breast, including the pectoral muscles and the glands found in the right
upon questions of law alone. We do not deem it necessary, therefore, to consider
armpit. She stayed in the hospital for a period of eight days, after which she was
appellee's additional argument, which was upheld by the trial court, that in signing
13

the application form in blank and leaving it to Edward A. Santos to fill (assuming Co., 51 Phil. 725, 732, this Court said, quoting from Joyce, The Law of Insurance, 2nd
that to be the truth) the insured in effect made Santos her agent for that purpose ed., Vol. 3:
and consequently was responsible for the errors in the entries made by him in that
capacity. "The basis of the rule vitiating the contract in cases of concealment is that
it misleads or deceives the insurer into accepting the risk, or accepting it at
In the application for insurance signed by the insured in this case, she agreed to the rate of premium agreed upon. The insurer, relying upon the belief that
submit to a medical examination by a duly appointed examiner of appellee if in the the assured will disclose every material fact within his actual or presumed
latter's opinion such examination was necessary as further evidence of insurability. knowledge, is misled into a belief that the circumstance withheld does not
In not asking her to submit to a medical examination, appellants maintain, appellee exist, and he is thereby induced to estimate the risk upon a false basis that
was guilty of negligence, which precluded it from finding about her actual state of it does not exist."
health. No such negligence can be imputed to appellee. It was precisely because the
insured had given herself a clean bill of health that appellee no longer considered The judgment appealed from, dismissing the complaint and awarding the return to
an actual medical checkup necessary. appellants of the premium already paid, with interest at 6% up to January 29, 1959,
affirmed, with costs against appellants.
Appellants also contend there was no fraudulent concealment of the truth
inasmuch as the insured herself did not know, since her doctor never told her, that
the disease for which she had been operated on was cancer. In the first place the
concealment of the fact of the operation itself was fraudulent, as there could not
have been any mistake about it, no matter what the ailment. Secondly, in order to
avoid a policy it is not necessary to show actual fraud on the part of the insured. In
the case of Kasprzyk v. Metropolitan Insurance Co., 140 N.Y.S. 211, 214, it was held:

Moreover, if it were the law that an insurance company could not depend
a policy on the ground of misrepresentation, unless it could show actual
knowledge on the part of the applicant that the statements were false,
then it is plain that it would be impossible for it to protect itself and its
honest policyholders against fraudulent and improper claims. It would be
wholly at the mercy of any one who wished to apply for insurance, as it
would be impossible to show actual fraud except in the extremest cases. It
could not rely on an application as containing information on which it
could act. There would be no incentive to an applicant to tell the truth.

Wherefore, the parties respectfully pray that the foregoing stipulation of


facts be admitted and approved by this Honorable Court, without
prejudice to the parties adducing other evidence to prove their case not
covered by this stipulation of facts. 1äwphï1.ñët

In this jurisdiction a concealment, whether intentional or unintentional, entitles the


insurer to rescind the contract of insurance, concealment being defined as
"negligence to communicate that which a party knows and ought to communicate"
(Sections 24 & 26, Act No. 2427). In the case of Argente v. West Coast Life Insurance
14

Saturnino v. Philamlife - False Representation The question to determine is: Are the facts then falsely represented material? The
Insurance Law provides that “materiality is to be determined not by the event, but
7 SCRA 316
solely by the probable and reasonable influence of the facts upon the party to
whom the communication is due, in forming his estimate of the proposed contract,
Facts:
or making his inquiries.
> 2 months prior to the insurance of the policy, Saturnino was operated on for
The contention of appellants is that the facts subject of the representation were not
cancer, involving complete removal of the right breast, including the pectoral
material in view of the non-medical nature of the insurance applied for, which does
muscles and the glands, found in the right armpit.
away with the usual requirement of medical examination before the policy is
> Notwithstanding the fact of her operation, Saturnino did not make a disclosure issued. The contention is without merit. If anything, the waiver of medical
thereof in her application for insurance. examination renders even more material the information required of the applicant
concerning previous condition of health and diseases suffered, for such information
> She stated therein that she did not have, nor had she ever had, among others
necessarily constitutes an important factor which the insurer takes into
listed in the application, cancer or other tumors; that she had not consulted any
consideration in deciding whether to issue the policy or not.
physician, undergone any operation or suffered any injury within the preceding 5
years. Appellants also contend that there was no fraudulent concealment of the truth
inasmuch as the insured herself did not know, since her doctor never told her, that
> She also stated that she had never been treated for, nor did she ever have any
the disease for which she had been operated on was cancer. In the first place,
illness or disease peculiar to her sex, particularly of the breast, ovaries, uterus and
concealment of the fact of the operation itself was fraudulent, as there could not
menstrual disorders.
have been any mistake about it, no matter what the ailment.
> The application also recited that the declarations of Saturnino constituted a
further basis for the issuance of the policy.
Secondly, in order to avoid a policy, it is not necessary to show actual fraud on the
Issue: part of the insured. In this jurisdiction, concealment, whether intentional or
Whether or not the insured made such false representation of material facts as to unintentional entitled the insurer to rescind the contract of insurance, concealment
avoid the policy. being defined as “negligence to communicate that which a party knows and ought
to communicate.” The basis of the rule vitiating the contract in cases of
Held: concealment is that it misleads or deceives the insurer into accepting the risk, or
accepting it at a rate of premium agreed upon. The insurer, relying upon the belief
YES.
that the insured will disclose every material fact within his actual or presumed
There can be no dispute that the information given by her in the application for knowledge, is misled into a belief that the circumstances withheld does not exist,
insurance was false, namely, that she never had cancer or tumors or consulted any and he is thereby induced to estimate the risk upon a false basis that it does not
physician or undergone any operation within the preceding period of 5 years. exist.
15

SCRA 316 SATURNINO V. PHILAMLIFE -FALSE REPRESENTATION 7 SCRA 316 In this jurisdiction,concealment, whether intentional orunintentional entitled the
insurer to rescindthe contract of insurance, concealment being defined as
Facts:> 2 months prior to the insurance of thepolicy, Saturnino was operated on ―negligence to communicate that which a party knows and
forcancer, involving complete removal of theright breast, including the pectoral ought to communicate.‖ The basis of therule vitiating the contract in cases
musclesand the glands, found in the right armpit.> Notwithstanding the fact of her ofconcealment is that it misleads or deceivesthe insurer into accepting the risk,
operation,Saturnino did not make a disclosure thereofin her application for oraccepting it at a rate of premium agreedupon. The insurer,
insurance.> She stated therein that she did not have,nor had she ever had, among relying upon the beliefthat the insured will disclose every materialfact within his
others listedin the application, cancer or other tumors;that she had not consulted actual or presumed knowledge, is misled into a belief that thecircumstances withheld does not
any physician,undergone any operation or suffered anyinjury within the preceding 5 exist, andhe is thereby induced to estimate the riskupon a false basis that it does not exist.
years.> She also stated that she had never beentreated for, nor did she ever have
anyillness or disease peculiar to her sex,particularly of the breast, ovaries, SCRA 316 – Mercantile Law – Insurance Law – Representation – Concealment –
uterusand menstrual disorders.> The application also recited that thedeclarations
Misrepresentation – Fraud
of Saturnino constituted afurther basis for the issuance of the policy.Issue:Whether
or not the insured made such falserepresentation of material facts as to avoidthe In September 1957, Estefania Saturnino was operated for cancer in which her right
policy.Held: YES.There can be no dispute that the informationgiven by her in the breast was removed. She was advised by her surgeon that she’s not totally cured
application for insurancewas false, namely, that she never hadcancer or tumors or
because her cancer was malignant. In November 1957, she applied for an insurance
consulted any physicianor undergone any operation within thepreceding period of
5 years.The question to determine is: Are the policy under Philamlife (Philippine American Life Insurance Company). She did not
factsthen falsely represented material? TheInsurance Law provides disclose the fact that she was operated nor did she disclose any medical histories.
that―materiality is to be determined not by the event, butsolely by the probable Philamlife, upon seeing the clean bill of health from Estefania waived its right to
and reasonableinfluence of the facts upon the party towhom the communication is have Estefania undergo a medical checkup. In September 1958, Estefania died of
due, in forminghis estimate of the proposed contract, ormaking his inquiries.The
pneumonia secondary to influenza. Her heirs now seek to enforce the insurance
contention of appellants is that the factssubject of the representation were
claim.
notmaterial in view of the non-medical nature ofthe insurance applied for, which
does awaywith the usual requirement of medicalexamination before the policy is ISSUE: Whether or not Saturnino is entitled to the insurance claim.
issued.The contention is without merit. If anything,the waiver of medical
examination renderseven more material the information requiredof the applicant HELD: No. The concealment of the fact of the operation is fraudulent. Even if, as
concerning previouscondition of health and diseases suffered,for such information argued by the heirs, Estefania never knew she was operated for cancer, there is still
necessarily constitutesan important factor which the insurer takesinto fraud in the concealment no matter what the ailment she was operated for. Note
consideration in deciding whether toissue the policy or also that in order to avoid a policy, it is not necessary that actual fraud be
not. Appellants also contend that there was nofraudulent concealment of the
established otherwise insurance companies will be at the mercy of any one seeking
truthinasmuch as the insured herself did notknow, since her doctor never told her,
thatthe disease for which she had beenoperated on was cancer. In the first insurance.
place,concealment of the fact of the operationitself was fraudulent, as there could In this jurisdiction a concealment, whether intentional or unintentional, entitles the
nothave been any mistake about it, no matterwhat the ailment.Secondly, in order
insurer to rescind the contract of insurance, concealment being defined as
to avoid a policy, it is notnecessary to show actual fraud on the partof the insured.
“negligence to communicate that which a party knows and ought to communicate.”
16

Also, the fact that Philamlife waived its right to have Estefania undergo a medical
examination is not negligence. Because of Estefania’s concealment, Philamlife
considered medical checkup to be no longer necessary. Had Philamlife been
informed of her operation, she would have been made to undergo medical checkup
to determine her insurability.
17

FIRST DIVISION letter stated that the said life insurance application for 20-year endowment plan is
not available for minors below seven years old, but Pacific Life can consider the
G.R. No. L-31845 April 30, 1979 same under the Juvenile Triple Action Plan, and advised that if the offer is
acceptable, the Juvenile Non-Medical Declaration be sent to the company.
GREAT PACIFIC LIFE ASSURANCE COMPANY, petitioner,
vs. The non-acceptance of the insurance plan by Pacific Life was allegedly not
HONORABLE COURT OF APPEALS, respondents. communicated by petitioner Mondragon to private respondent Ngo Hing. Instead,
on May 6, 1957, Mondragon wrote back Pacific Life again strongly recommending
G.R. No. L-31878 April 30, 1979 the approval of the 20-year endowment insurance plan to children, pointing out
that since 1954 the customers, especially the Chinese, were asking for such
coverage (Exhibit 4-M).
LAPULAPU D. MONDRAGON, petitioner,
vs.
HON. COURT OF APPEALS and NGO HING, respondents. It was when things were in such state that on May 28, 1957 Helen Go died of
influenza with complication of bronchopneumonia. Thereupon, private respondent
sought the payment of the proceeds of the insurance, but having failed in his effort,
DE CASTRO, J.:
he filed the action for the recovery of the same before the Court of First Instance of
Cebu, which rendered the adverse decision as earlier refered to against both
The two above-entitled cases were ordered consolidated by the Resolution of this
petitioners.
Court dated April 29, 1970, (Rollo, No. L-31878, p. 58), because the petitioners in
both cases seek similar relief, through these petitions for certiorari by way of
The decisive issues in these cases are: (1) whether the binding deposit receipt
appeal, from the amended decision of respondent Court of Appeals which affirmed
(Exhibit E) constituted a temporary contract of the life insurance in question; and
in toto the decision of the Court of First Instance of Cebu, ordering "the defendants
(2) whether private respondent Ngo Hing concealed the state of health and physical
(herein petitioners Great Pacific Ligfe Assurance Company and Mondragon) jointly
condition of Helen Go, which rendered void the aforesaid Exhibit E.
and severally to pay plaintiff (herein private respondent Ngo Hing) the amount of
P50,000.00 with interest at 6% from the date of the filing of the complaint, and the
sum of P1,077.75, without interest. 1. At the back of Exhibit E are condition precedents required before a deposit is
considered a BINDING RECEIPT. These conditions state that:
It appears that on March 14, 1957, private respondent Ngo Hing filed an application
with the Great Pacific Life Assurance Company (hereinafter referred to as Pacific A. If the Company or its agent, shan have received the premium
Life) for a twenty-year endownment policy in the amount of P50,000.00 on the life deposit ... and the insurance application, ON or PRIOR to the date
of his one-year old daughter Helen Go. Said respondent supplied the essential data of medical examination ... said insurance shan be in force and in
which petitioner Lapulapu D. Mondragon, Branch Manager of the Pacific Life in effect from the date of such medical examination, for such period
Cebu City wrote on the corresponding form in his own handwriting (Exhibit I-M). as is covered by the deposit ...,PROVIDED the company shall be
Mondragon finally type-wrote the data on the application form which was signed by satisfied that on said date the applicant was insurable on standard
private respondent Ngo Hing. The latter paid the annual premuim the sum of rates under its rule for the amount of insurance and the kind of
P1,077.75 going over to the Company, but he reatined the amount of P1,317.00 as policy requested in the application.
his commission for being a duly authorized agebt of Pacific Life. Upon the payment
of the insurance premuim, the binding deposit receipt (Exhibit E) was issued to D. If the Company does not accept the application on standard
private respondent Ngo Hing. Likewise, petitioner Mondragon handwrote at the rate for the amount of insurance and/or the kind of policy
bottom of the back page of the application form his strong recommendation for the requested in the application but issue, or offers to issue a policy
approval of the insurance application. Then on April 30, 1957, Mondragon received for a different plan and/or amount ..., the insurance shall not be in
a letter from Pacific Life disapproving the insurance application (Exhibit 3-M). The force and in effect until the applicant shall have accepted the
18

policy as issued or offered by the Company and shall have paid the ground that it is not offering the twenty-year endowment insurance policy to
full premium thereof. If the applicant does not accept the policy, children less than seven years of age. What it offered instead is another plan known
the deposit shall be refunded. as the Juvenile Triple Action, which private respondent failed to accept. In the
absence of a meeting of the minds between petitioner Pacific Life and private
E. If the applicant shall not have been insurable under Condition A respondent Ngo Hing over the 20-year endowment life insurance in the amount of
above, and the Company declines to approve the application the P50,000.00 in favor of the latter's one-year old daughter, and with the non-
insurance applied for shall not have been in force at any time and compliance of the abovequoted conditions stated in the disputed binding deposit
the sum paid be returned to the applicant upon the surrender of receipt, there could have been no insurance contract duly perfected between thenl
this receipt. (Emphasis Ours). Accordingly, the deposit paid by private respondent shall have to be refunded by
Pacific Life.
The aforequoted provisions printed on Exhibit E show that the binding deposit
receipt is intended to be merely a provisional or temporary insurance contract and As held in De Lim vs. Sun Life Assurance Company of Canada, supra, "a contract of
only upon compliance of the following conditions: (1) that the company shall be insurance, like other contracts, must be assented to by both parties either in person
satisfied that the applicant was insurable on standard rates; (2) that if the company or by their agents ... The contract, to be binding from the date of the application,
does not accept the application and offers to issue a policy for a different plan, the must have been a completed contract, one that leaves nothing to be dione, nothing
insurance contract shall not be binding until the applicant accepts the policy to be completed, nothing to be passed upon, or determined, before it shall take
offered; otherwise, the deposit shall be reftmded; and (3) that if the applicant is not effect. There can be no contract of insurance unless the minds of the parties have
ble according to the standard rates, and the company disapproves the application, met in agreement."
the insurance applied for shall not be in force at any time, and the premium paid
shall be returned to the applicant. We are not impressed with private respondent's contention that failure of
petitioner Mondragon to communicate to him the rejection of the insurance
Clearly implied from the aforesaid conditions is that the binding deposit receipt in application would not have any adverse effect on the allegedly perfected temporary
question is merely an acknowledgment, on behalf of the company, that the latter's contract (Respondent's Brief, pp. 13-14). In this first place, there was no contract
branch office had received from the applicant the insurance premium and had perfected between the parties who had no meeting of their minds. Private
accepted the application subject for processing by the insurance company; and that respondet, being an authorized insurance agent of Pacific Life at Cebu branch office,
the latter will either approve or reject the same on the basis of whether or not the is indubitably aware that said company does not offer the life insurance applied for.
applicant is "insurable on standard rates." Since petitioner Pacific Life disapproved When he filed the insurance application in dispute, private respondent was,
the insurance application of respondent Ngo Hing, the binding deposit receipt in therefore, only taking the chance that Pacific Life will approve the recommendation
question had never become in force at any time. of Mondragon for the acceptance and approval of the application in question along
with his proposal that the insurance company starts to offer the 20-year
Upon this premise, the binding deposit receipt (Exhibit E) is, manifestly, merely endowment insurance plan for children less than seven years. Nonetheless, the
conditional and does not insure outright. As held by this Court, where an agreement record discloses that Pacific Life had rejected the proposal and recommendation.
is made between the applicant and the agent, no liability shall attach until the Secondly, having an insurable interest on the life of his one-year old daughter, aside
principal approves the risk and a receipt is given by the agent. The acceptance is from being an insurance agent and an offense associate of petitioner Mondragon,
merely conditional and is subordinated to the act of the company in approving or private respondent Ngo Hing must have known and followed the progress on the
rejecting the application. Thus, in life insurance, a "binding slip" or "binding receipt" processing of such application and could not pretend ignorance of the Company's
does not insure by itself (De Lim vs. Sun Life Assurance Company of Canada, 41 Phil. rejection of the 20-year endowment life insurance application.
264).
At this juncture, We find it fit to quote with approval, the very apt observation of
It bears repeating that through the intra-company communication of April 30, 1957 then Appellate Associate Justice Ruperto G. Martin who later came up to this Court,
(Exhibit 3-M), Pacific Life disapproved the insurance application in question on the
19

from his dissenting opinion to the amended decision of the respondent court which have verified the same and would have had no choice but to disapprove the
completely reversed the original decision, the following: application outright.

Of course, there is the insinuation that neither the memorandum of The contract of insurance is one of perfect good faith uberrima fides meaning good
rejection (Exhibit 3-M) nor the reply thereto of appellant Mondragon faith, absolute and perfect candor or openness and honesty; the absence of any
reiterating the desire for applicant's father to have the application concealment or demotion, however slight [Black's Law Dictionary, 2nd Edition], not
considered as one for a 20-year endowment plan was ever duly for the alone but equally so for the insurer (Field man's Insurance Co., Inc. vs. Vda
communicated to Ngo; Hing, father of the minor applicant. I am not quite de Songco, 25 SCRA 70). Concealment is a neglect to communicate that which a
conninced that this was so. Ngo Hing, as father of the applicant herself, partY knows aDd Ought to communicate (Section 25, Act No. 2427). Whether
was precisely the "underwriter who wrote this case" (Exhibit H-1). The intentional or unintentional the concealment entitles the insurer to rescind the
unchallenged statement of appellant Mondragon in his letter of May 6, contract of insurance (Section 26, Id.: Yu Pang Cheng vs. Court of Appeals, et al, 105
1957) (Exhibit 4-M), specifically admits that said Ngo Hing was "our Phil 930; Satumino vs. Philippine American Life Insurance Company, 7 SCRA 316).
associate" and that it was the latter who "insisted that the plan be placed Private respondent appears guilty thereof.
on the 20-year endowment plan." Under these circumstances, it is
inconceivable that the progress in the processing of the application was We are thus constrained to hold that no insurance contract was perfected between
not brought home to his knowledge. He must have been duly apprised of the parties with the noncompliance of the conditions provided in the binding
the rejection of the application for a 20-year endowment plan otherwise receipt, and concealment, as legally defined, having been comraitted by herein
Mondragon would not have asserted that it was Ngo Hing himself who private respondent.
insisted on the application as originally filed, thereby implictly declining the
offer to consider the application under the Juvenile Triple Action Plan. WHEREFORE, the decision appealed from is hereby set aside, and in lieu thereof,
Besides, the associate of Mondragon that he was, Ngo Hing should only be one is hereby entered absolving petitioners Lapulapu D. Mondragon and Great
presumed to know what kind of policies are available in the company for Pacific Life Assurance Company from their civil liabilities as found by respondent
minors below 7 years old. What he and Mondragon were apparently trying Court and ordering the aforesaid insurance company to reimburse the amount of
to do in the premises was merely to prod the company into going into the P1,077.75, without interest, to private respondent, Ngo Hing. Costs against private
business of issuing endowment policies for minors just as other insurance respondent.
companies allegedly do. Until such a definite policy is however, adopted by
the company, it can hardly be said that it could have been bound at all
SO ORDERED.
under the binding slip for a plan of insurance that it could not have, by
then issued at all. (Amended Decision, Rollo, pp- 52-53).

2. Relative to the second issue of alleged concealment. this Court is of the firm
belief that private respondent had deliberately concealed the state of health and
piysical condition of his daughter Helen Go. Wher private regpondeit supplied the
required essential data for the insurance application form, he was fully aware that
his one-year old daughter is typically a mongoloid child. Such a congenital physical
defect could never be ensconced nor disguished. Nonetheless, private respondent,
in apparent bad faith, withheld the fact materal to the risk to be assumed by the
insurance compary. As an insurance agent of Pacific Life, he ought to know, as he
surely must have known. his duty and responsibility to such a material fact. Had he
diamond said significant fact in the insurance application fom Pacific Life would
20

Grepalife v. CA Whether or not the binding deposit receipt, constituted a temporary contract of
life insurance.
89 SCRA 543
Held:
Facts:
NO.
> On March 14, 1957, respondent Ngo Hing filed an application with Grepalife for a
The binding receipt in question was merely an acknowledgement on behalf of the
20-yr endowment policy for 50T on the life of his one year old daughter Helen Go.
company, that the latter’s branch office had received from the applicant, the
> All the essential data regarding Helen was supplied by Ngo to Lapu-Lapu insurance premium and had accepted the application subject for processing by the
Mondragon, the branch manager of Grepalife-Cebu. Mondragon then typed the insurance company, and that the latter will either approve or reject the same on
data on the application form which was later signed by Ngo. the basis of whether or not the applicant is insurable on standard rates.

> Ngo then paid the insurance premium and a binding deposit receipt was issued to Since Grepalife disapproved the insurance application of Ngo, the binding deposit
him. The binding receipt contained the following provision: “If the applicant shall receipt had never became on force at any time, pursuant to par. E of the said
not have been insurable xxx and the Company declines to approve the application, receipt. A binding receipt is manifestly merely conditional and does NOT insure
the insurance applied for shall not have been in force at any time and the sum paid outright. Where an agreement is made between the applicant and the agent, NO
shall be returned to the applicant upon the surrender of this receipt.” liability shall attach until the principal approves the risk and a receipt is given by the
agent.
> Mondragon wrote on the bottom of the application form his strong
recommendation for the approval of the insurance application. The acceptance is merely conditional, and is subordinated to the act of the
company in approving or rejecting the application. Thus in life insurance, a binding
> On Apr 30, 1957, Mondragon received a letter from Grepalife Main office
slip or binding receipt does NOT insure by itself.
disapproving the insurance application of Ngo for the simple reason that the 20yr
endowment plan is not available for minors below 7 yrs old.

> Mondragon wrote back the main office again strongly recommending the Great Pacific Life Assurance Co. v Court of Appeals89 SCRA 543April 30,
approval of the endowment plan on the life of Helen, adding that Grepalife was the 1979Facts:Respondent Ngo Hing filed an application with petitioner Great Pacific
only insurance company NOT selling endowment plans to children. Life Assurance Company(Pacific Life) for a twenty-year endowment policy in the life
of Helen Go, his one year olddaughter. Petitioner Lapulapu D. Mondragon, the
> On may 1957, Helen died of influenza with complication of broncho pneumonia. branch manager, prepared application formusing the essential data supplied by
Ngo filed a claim with Gepalife, but the latter denied liability on the ground that respondent. The latter paid the annual premium andMondragon retained a portion
there was no contract between the insurer and the insured and a binding receipt is of it as his commission. The binding deposit receipt was issued
NOT evidence of such contract. torespondent.Mondragon wrote his strong recommendation for the approval of the
insurance application.However, Pacific Life disapproved the application since
Issue: the plan was not available for minorsbelow 7 years old but it can consider the same
under another plan. The non-acceptance of theinsurance plan was allegedly not
communicated by Mondragon to respondent. Mondragon againasserted his strong
21

recommendation.Helen Go died of influenza. Thereupon, respondent sought the


payment of the proceeds of theinsurance, but having failed in his effort, he filed an
action for the recovery of the same. Hencethe case at bar.Issue:Whether or not the
insurance contract has been perfected on the ground that a binding receipthas
been issued?Held:NO, it was not perfected. The binding deposit receipt is merely an
acknowledgement, on behalf of the company, that the latter’s branch office had
received from the applicant the insurancepremium and had accepted the
application subject for processing by the insurance company; andthat the latter will
either approve or reject the same on the basis of whether or not the applicant
isinsurable on standard rates.The binding deposit receipt is merely conditional and
does not insure outright. Where anagreement is made between the applicant and
the agent, no liability shall attach until the principalapproves the risk and a receipt
is given by the agent. The acceptance is merely conditional, andis subordinated to
the act of the company in approving or rejecting the application. Thus, in
lifeinsurance, a ‘binding slip’ or ‘binding receipt’ does not insure by itself.
22

EN BANC the Administrative Code, which became effective on July 1, 1916, and it has been
suggested that the judgment convicting and sentencing the accused under the
G.R. No. L-11555 January 6, 1917 provisions of that statute should not be sustained, and that the repeal of the
statute should be held to have the effect of remitting and extinguishing the criminal
THE UNITED STATES, plaintiff-appellee, responsibility of the accused incurred under the provisions of the repealed law prior
vs. to the enactment of the Administrative Code. We cannot agree with the proposition
GABINO SOLIMAN, defendant-appellant. thus stated.

Francisco Sevilla for appellant. In the case of United States vs. Cuna (12 Phil. Rep., 241), we held as follows:
Attorney-General Avanceña for appellee.
The rule of interpretation of English and American common law, by virtue
CARSON, J.: of which the repeal of a law prescribing penalties is held to have the effect
of remitting or extinguishing any penalty, loss of rights or responsibility
incurred under such law, as to all persons who have not been convicted
The evidence of record conclusively discloses that the defendant and appellant in
and sentenced under the provisions of such law prior to the enactment of
this case, Gabino Soliman, testifying in his on behalf in the course of another
the repealing law, is not and has not been the accepted doctrine in these
criminal case in which he, with several others, was charged withestafa, swore
Islands.
falsely to certain material allegations of fact.

Where an Act of the Commission or of the Philippine Legislature which


On that occasion he testified falsely that a sworn statement offered in evidence in
penalizes an offense, such repeal does not have the effect of thereafter
support of the charge of estafa, which was in effect an extrajudicial confession of
depriving the courts of jurisdiction to try, convict and sentence offenders
his guilt, had not been executed voluntarily, and that its execution had not been
charged with violations of the old law prior to its repeal.
procured by the police by the use of force, intimidation and prolonged torture.

A question does arise, however, as to the penalty which should be impose upon the
The trial judge who presided in the former case acquitted the accused on the
convict.
ground that there was room for reasonable doubt as to whether the extrajudicial
confession had been made voluntarily, and his action in this regard clearly
establishes the materiality of the false testimony submitted in that case; moreover, If the repealing statute provides or has the effect of providing new penalties for the
the materiality of the evidence is manifest without considering the judgment in the commission of the acts penalized under the repealed statute, should the penalty be
case in which it was submitted, since, if accepted as true, this false testimony imposed in accordance with the old or the new statute?
necessarily had the effect of rendering wholly incompetent the evidence as to the
extrajudicial confession which otherwise would almost conclusively sustain and Article 1 of the Penal Code in force in these Islands defines crimes and
necessitate a conviction. (U. S. vs.Estraña, 16 Phil. Rep., 520.) misdemeanors as voluntary acts or omissions penalized by law; and complementary
to this provision, article 21 provides that no crime or misdemeanor shall be
There can be no doubt that the accused was guilty of the crime of perjury as punished with a penalty which has not been prescribed by law prior to its
defined and penalized in section 3 of Act No. 1697 and that the sentence of six commission. In accordance with these provisions the question whether an act is
months' imprisonment and P300 fine imposed by the trial judge was correctly punishable or not depends upon the question whether or not at the time of its
imposed under the provisions of that statute. commission, there was a law in force which penalized it; this rule being modified,
however, by article 22 of the same code, which provides that penal laws shall have
a retroactive effect in so far as they favor persons convicted of a crime or
It appears however that since judgment was entered in this case on November 23,
misdemeanor.
1915, section 3 of Act No. 1697 has been expressly repealed by the enactment of
23

The courts of Spain and the learned commentators on Spanish law have construed We conclude therefore that in any case in which a statute prescribing a penalty for
these provisions to mean that such penal laws are to be given a retroactive effect the commission of a specific offense is repealed, and in which the new statute
only in so far as they favor the defendant charged with a crime or a misdemeanor, provides new and distinct penalties for the commission of such offense, the penalty
and that, when a penal law is enacted repealing a prior law, such repeal does not which must be imposed on one who committed the offense prior to the enactment
have the effect of relieving an offender in whole or in part of penalties already of the repealing statute is that one which is more favorable to the convict. (U.
incurred under the old law, unless the new law favors the defendant by diminishing S. vs. Cuna, 12 Phil. Rep., 241.)
the penalty or doing away with it altogether, and then only to the extent to which
the new law is favorable to the offender. In other words, that the enactment of new It seems important, then, to determine whether the repeal of section 3 of Act No.
penal laws, notwithstanding the fact that they contain general repealing clauses, 1697 by the enactment of the Administrative Code had the effect of providing new
doe not deprive the courts of jurisdiction to try, convict and sentence persons and distinct penalties for the commission of the crime of perjury, and whether the
charged with violations of the old law prior to the date when the repealing law goes new penalties are or are not more favorable to the convict in the case at bar than
into effect, unless the new law wholly fails to penalties the acts which constituted those imposed by the trial judge.
the offense defined and penalized in the repealed law.
Section 3 of Act No. 1697, which defined and penalized the crime of perjury,
Thus Pacheco, commenting upon the new Penal Code of 1848-1850, of which article repealed the provisions of the Penal Code defining and penalizing the crime of
506 provided that all general penal laws were repealed by its publication, says: perjury, not expressly, but by implication, and we are of opinion that the repeal of
Act No. 1697 revived those provisions of the code. (U. S. vs. Concepcion, 13 Phil.
At this time when the Penal Code is being put into effect and given force, Rep., 424; U. S. vs.Estraña, 16 Phil. Rep., 520.)
we have in fact two criminal laws in Spain, and close attention is necessary
to apply them properly. There may be prosecutions which it is necessary to In the absence of the most express language to the contrary it will not be presumed
dismiss, as, for example, those for sodomy; others which it may be that it was the intention of the legislator to let false swearing as to a material
necessary to decide in conformity with the provisions of the new codes, as, matter in a court of justice go unpunished, and such would be the effect of the
for example, those for carrying concealed weapons; and others which must repeal of section 3 of Act No. 1697, unless we held that the repeal had the effect of
be judged in accordance with the old provisions, as, for example. many reviving the old statute.
cases of robbery. The rules of procedure in one or other manner being
furnished us by the former article (article 19 of the Penal Code of Spain At the common law the repeal of a repealing act revived the former act (6 Co., 199;
identical with article 21 of the Penal Code of the Philippines), and the 1 Gray, 163; 7 W. & S., 263; 2 Blackstone, 32; 54 N. J. L. J., 175); and the Supreme
present article (article 20 of the Penal Code of Spain and article 22 of the Court of the United States has held that the repeal of a repealing law has this effect,
Philippine Code). Has the code increased the penalty? Then it is not unless the language of the repealing statute or some general statute provides
applicable to crimes committed prior to its enactment. Has it extinguished otherwise. (U. S. vs. Otis, 120 U. S., 52 [115].)
or diminished them? Then it is clearly applicable to them. (1 Pacheco, 296.)
Manifestly, with this rule in mind, section 12 of the Administrative Code (Act No.
And a similar construction was placed upon the provisions of the Penal Code of 2657) which is found in Article III, [Chapter I] dealing with the form and effect of
1870 by the supreme court of Spain. Article 626 of this code (which is substantially laws in general, provides that "when a law which expressly repeals a prior law is
identical with article 506 of the Penal Code of 1848 and article 611 of the Penal itself repealed the law first repealed shall not be thereby revived unless expressly
Code of the Philippine Islands) repealed all general penal laws prior to its so provided." From which it may fairly be inferred that the old rule continues in
promulgation, but the court held that, where a crime was committed prior to the force where a law which repeals a prior law, not expressly but by implication, it
publication of the reformed code, the penalty prescribed by the code of 1850 (the itself repealed; and that in such cases the repeal of the repealing law revives the
code prior to that of 1870) being more favorable to the accused, that must be prior law, unless the language of the repealing statute provides otherwise.
applied. (Decision of the supreme court of Spain, 17th of January, 1873.)
24

Applying this rule, we conclude that the express repeal of section 3 of Act No. 1697 in his own cause, like any other witness, he incurs the penalty by which false
by the enactment of the Administrative Code (Act No. 2657) revived the provisions testimony" in criminal matters "is repressed and punished."
of the Penal Code touching perjury, which were themselves repealed, not expressly
but by implication, by the enactment of Act No. 1697. It has been suggested that such a ruling will have a tendency to expose accused
persons to vexatious criminal prosecutions by prosecuting officers, who, having
A comparison of the penalties prescribed in the Penal Code for the commission of failed to secure a conviction on the original charge, may be disposed to institute
the acts of which the accused in the case at bar was convicted, giving him as we criminal prosecutions for perjury from a vindictive unwillingness to let the
should the benefit of the provisions of Act No. 2142, discloses that the penalty defendant escape scot free from the meshes of the law. It is said also that the fear
prescribed therein is less than that imposed upon the appellant under the of subsequent prosecution for perjury will tend to embarrass accused persons in
provisions of section 3 of Act No. 1697, and we conclude from what has been said their efforts to defend themselves by testifying in their own behalf. But similar
already that the penalty imposed by the court below should be revoked and that in objections may be advanced against the prosecution of any of the witnesses called
lieu thereof the penalty prescribed in the Penal Code should be imposed upon the for the defense on charges of perjury, and it must not be forgotten that the right of
convict. an accused person to testify under oath in his own behalf is secured to him, not that
he may be enabled to introduce false testimony into the record, but to enable him
A question has been raised as to whether, admitting that the provisions of the Penal to spread upon the record the truth as to any matter within his knowledge which
Code touching perjury have been revived, the accused can be convicted and will tend to establish his innocence.
penalized thereunder, it appearing that at the time when he testified falsely he was
testifying in his own behalf in a criminal case in which he himself was the accused, Of course much must be left to the good sense and sound judgment of the
on trial for the commission of a grave offense. prosecuting officer in determining whether a prosecution for perjury should be
instituted against an accused person whose testimony in his own behalf would
In the case of United States vs. Gutierrez (12 Phil. Rep., 529), we said, speaking seem to be perjured.
through Chief Justice Arellano, that, "Perjury committed by a party in his own cause
would not be punishable under Spanish legislation, because in said legislation no Due regard for the situation in which an accused person finds himself when
one was a witness in his own cause, and could not therefore become guilty of giving testifying in his own behalf in a criminal proceeding will restrain a prudent
false testimony in a civil cause in which he was either the plaintiff or the defendant; prosecuting officer from the filing of charges of perjury in every case in which he
but under the procedure in force by virtue of Act No. 190, a party to a suit may may have reason to believe that the accused has not adhered strictly to the truth, in
testify in his own behalf, and if he declares falsely under oath as a witness in his his anxiety to shield himself from punishment. But when, as in the case at bar, an
own cause, like any other witness, he incurs the penalty by which false testimony in accused person voluntarily goes upon the witness stand and falsely imputes some
civil matters is repressed and punished. This court has so held, it being a settled other person the commission of a grave offense, it would seem to be highly proper
rule, that the false testimony given by a litigant as a witness constitutes the crime of that he should be called to account in a criminal action for perjury upon the
giving false testimony inasmuch as such a declaration, according to the new laws in complaint of the person against whom such false charges are made.
force, may determine a judgment in his favor and to the prejudice of the adverse
party, and that a litigant who, in sworn testimony given by him as a witness in a civil Article 319 of the Penal Code is as follows:
cause, shall pervert the truth and give false testimony, incurs as such witness the
penalties imposed by article 321 of the Penal Code." Any person who shall give false testimony in favor of a defendant in a
criminal case shall suffer a penalty ranging from arresto mayor in its
Analogous reasoning leads to a like conclusion as to the criminal liability for perjury maximum degree to prision correccional in its medium degree and a fine of
of a defendant in a criminal case testifying falsely in his own behalf. Under the not less than three hundred and seventy-five and not more than three
provisions of General Orders No. 58 an accused person may, if he so desires, testify thousand seven hundred and fiftypesetas, if the case were for a felony,
under oath in his own behalf, and in that event, "if he declares falsely as a witness and the penalty of arresto mayor if it were for a misdemeanor.
25

We conclude that the judgment of conviction entered in the court below should be Yes.
affirmed but that the sentence imposed therein should be reversed, and that giving
the accused the benefit of the provisions of Act No. 2142, a penalty of 4 months The insurer is once again given two years from the date of reinstatement to
and 1 day of arresto mayor and a fine of P75 with subsidiary imprisonment as investigate into the veracity of the facts represented by the insured in the
prescribed by law should be imposed upon him in lieu of that imposed by the trial
application for reinstatement. When US life sought to rescind the contract on the
judge, with the costs of this instance de officio. So ordered.
ground of concealment/misrepresentation, two years had not yet elapsed. Hence,
Soliman v. US Life- Rescind Contract of Insurance the contract can still be rescinded.

104 PHIL 1046

Facts:

> US Life issued a 20 yr endowment life policy on the joint lives of Patricio Soliman
and his wife Rosario, each of them being the beneficiary of the other.

> In Mar. 1949, the spouses were informed that the premium for Jan 1949 was still
unpaid notwithstanding that the 31-day grace period has already expired, and they
were furnished at the same time long-form health certificates for the reinstatement
of the policies.

> In Apr 1949, they submitted the certificates and paid the premiums.

> In Jan. 1950, Rosario died of acute dilation of the heart, and thereafter, Patricio
filed a claim for the proceeds of the insurance.

> US life denied the claim and filed for the rescission of the contract on the ground
that the certificates failed to disclose that Rosario had been suffering from
bronchial asthma for 3 years prior to their submission.

Issue:

Whether or not the contract can still be rescinded.

Held:
26

EN BANC The face of the policy bore the annotation: "Co-Insurance Declared — NIL"

G.R. No. L-14373 January 30, 1960 It is undenied that Ng Hua had obtained fire insurance on the same goods, for the
same period of time, in the amount of P20,000.00 from General Indemnity Co.
GENERAL INSURANCE AND SURETY CORPORATION, petitioner, However, the Court of Appeals referring to the annotation and overruling the
vs. defense, held that there was no violation of the above clause, inasmuch as "co-
NG HUA, respondent. insurance exists when a condition of the policy requires the insured to bear ratable
proportion of the loss when the value of the insured property exceeds the face
BENGZON, J.: value of the policy," hence there is no co-insurance here.

Suit to recover on a fire insurance policy. The insurer presented several defenses in Discussion — Undoubtedly, co-insurance exists under the condition described by
the Manila court of first instance. After trial, it was required to pay. the appellate court. But that isone kind of co-insurance. It is not the only situation
where co-insurance exists. Other insurers of the same property against the same
hazard are sometimes referred as co-insurers and the ensuing combination as co-
On appeal to the Courts of Appeal, the judgment was affirmed.
insurance.1 And considering the terms of the policy which required the insured to
declare other insurances, the statement in question must be deemed to be a
This is now a revision on certiorari, upon the insurer's insistence on two of its main
statement (warranty) binding on both insurer and insured, that there were no other
defenses: prescription and breach of warranty.
insurance on the property. Remember it runs "Co-Insurance declared"; emphasis on
the last word. If "Co-Insurance" means that the Court of Appeals says, the
The principal of facts on which adjudication may rest are these: annotation served no purpose. It would even be contrary to the policy itself, which
in its clause No. 17 made the insured a co-insurer for the excess of the value of the
On April 15, 1952, the defendant General Insurance and Surety Corporation issued property over the amount of the policy.
its insurance Policy No. 471, insuring against fire, for one year, the stock in trade of
the Central Pomade Factory owned by Ng Hua, the court insured. The next day, the The annotation then, must be deemed to be a warranty that the property was not
Pomade factory building burned, resulting in destruction by fire of the insured insured by any other policy. Violation thereof entitles the insurer to rescind. (Sec.
properties. Ng Hua claimed indemnity from the insurer. The policy covered 69. Insurance Act) Such misrepresentation is fatal in the light of our views in Santa
damages up to P10,000.00; but after some negotiations and upon suggestion of the Ana vs. Commercial Union Assurance Company, Ltd., 55 Phil., 329. The materiality
Manila Adjustment Company, he reduced the claim of P5,000.00. Nevertheless, the of non-disclosure of other insurance policies is not open to doubt.
defendant insurer refused to pay for various reasons, namely (a) action was not
filed in time; (b) violation of warranty; (c) submission of fraudulent claim; and (f)
Furthermore, even if the annotations were overlooked, the defendant insurer
failure to pay the premium.
would still be free from liability because there is no question that the policy issued
by General Indemnity had not been stated in nor endorsed onPolicy No. 471 of
The aforesaid Policy No. 471 contains this stipulation on the back thereof;. defendant. And as stipulated in the above-quoted provisions of such policy "all
benefit under this policy shall be forfeited."2
3. The insured shall give notice to the company of any insurance or
insurances already affected, or which may subsequently be effected, To avoid the dissastrous effect of the misrepresentation or concealment of the
covering any of the property hereby insured, and unless such notice be other insurance policy, Ng Hua alleges "actual knowledge" on the part of General
given and the particulars of such insurance or insurances be stated in or insurance of the fact that he had taken out additional insurance with General
endorsed on this Policy by or on behalf of the Company before the Indemnity. He does not say when such knowledge was acquired or imparted. If
occurrence of any loss or damage, all benefits under the policy shall be General Insurance know before issuing its policy or before the fire, such knowledge
forfeited. (Emphasis ours.) might overcome the insurer's defense.3However, the Court of Appeals found no
27

evidence of such knowledge. We have read the pages of the stenographic notes > Subsequently, the claim of Ng Hua for the proceeds was denied by General since
cited by Ng Hua and we all gather is evidence of the existence of the Insurance
it discovered that Ng Hua had obtained an insurance from General Indemnity for
General Indemnity Company. As to knowledge of General Insurance before issuance
of its policy or the fire, there was none. the same goods and for the same period of time.

Indeed, this concealment and violation was expressly set up as a special defense in Issue:
the answer. Yet plaintiff did not, in avoidance, reply nor assert such knowledge. And
it is doubtful whether the evidence on the point would be admissible under the Whether or not General Insurance can refuse to pay the proceeds.
pleadings. (See Rule 11, sec. 1.)
Held:
All the above considerations lead to the conclusion that the defendant insurer
Yes.
successfully established its defense of warranty breach or concealment of the other
insurance and/or violation of the provision of the policy above-mentioned. Violation of the statement which is to be considered a warranty entitles the insurer
to rescind the contract of insurance. Such misrepresentation is fatal.
Having reached the conclusion, we deem it unnecessary to discuss the other
defenses.

Wherefore, the judgment under review will be revoked, and the defendant insurer
(herein petitioner) acquitted from all the liability under the policy. Costs against
respondent. So ordered.

Gen. Insurance & Surety Corp v. NG Hua - Misrepresentation

106 PHIL 1117

Facts:

> In 1952, General issued a fire policy to Ng Hua to cover the contents of the
Central Pomade Factory owned by him.

> There was a provision in the policy that should there be any insurance already
effected or to be subsequently procured, the insured shall give notice to the
insurer.

> Ng Hua declared that there was non. The very next day, the building and the
goods stored therein burned.
28

THIRD DIVISION deceased's insurance application had been approved on the basis of his medical
declaration. 8 She explained that as a rule, medical examinations are required only in
G.R. No. 92492 June 17, 1993 cases where the applicant has indicated in his application for insurance coverage
that he has previously undergone medical consultation and hospitalization. 9
THELMA VDA. DE CANILANG, petitioner,
vs. In a decision dated 5 November 1985, Insurance Commissioner Armando Ansaldo
HON. COURT OF APPEALS and GREAT PACIFIC LIFE ASSURANCE ordered Great Pacific to pay P19,700 plus legal interest and P2,000.00 as attorney's
CORPORATION, respondents. fees after holding that:

Simeon C. Sato for petitioner. 1. the ailment of Jaime Canilang was not so serious that, even if it had been
disclosed, it would not have affected Great Pacific's decision to insure him;
FELICIANO, J.:
2. Great Pacific had waived its right to inquire into the health condition of
On 18 June 1982, Jaime Canilang consulted Dr. Wilfredo B. Claudio and was the applicant by the issuance of the policy despite the lack of answers to
diagnosed as suffering from "sinus tachycardia." The doctor prescribed the following "some of the pertinent questions" in the insurance application;
fro him: Trazepam, a tranquilizer; and Aptin, a beta-blocker drug. Mr. Canilang
consulted the same doctor again on 3 August 1982 and this time was found to have 3. there was no intentional concealment on the part of the insured Jaime
"acute bronchitis." Canilang as he had thought that he was merely suffering from a minor
ailment and simple cold; 10 and
On next day, 4 August 1982, Jaime Canilang applied for a "non-medical" insurance
policy with respondent Great Pacific Life Assurance Company ("Great Pacific") 4. Batas Pambansa Blg. 847 which voids an insurance contract, whether or
naming his wife, Thelma Canilang, as his beneficiary. 1 Jaime Canilang was issued not concealment was intentionally made, was not applicable to Canilang's
ordinary life insurance Policy No. 345163, with the face value of P19,700, effective case as that law became effective only on 1 June 1985.
as of 9 August 1982.
On appeal by Great Pacific, the Court of Appeals reversed and set aside the decision
On 5 August 1983, Jaime Canilang died of "congestive heart failure," "anemia," and of the Insurance Commissioner and dismissed Thelma Canilang's complaint and
"chronic anemia." 2 Petitioner, widow and beneficiary of the insured, filed a claim Great Pacific's counterclaim. The Court of Appealed found that the use of the word
with Great Pacific which the insurer denied on 5 December 1983 upon the ground "intentionally" by the Insurance Commissioner in defining and resolving the issue
that the insured had concealed material information from it. agreed upon by the parties at pre-trial before the Insurance Commissioner was not
supported by the evidence; that the issue agreed upon by the parties had been
Petitioner then filed a complaint against Great Pacific with the Insurance whether the deceased insured, Jaime Canilang, made a material concealment as the
Commission for recovery of the insurance proceeds. During the hearing called by the state of his health at the time of the filing of insurance application, justifying
Insurance Commissioner, petitioner testified that she was not aware of any serious respondent's denial of the claim. The Court of Appeals also found that the failure of
illness suffered by her late husband 3 and that, as far as she knew, her husband had Jaime Canilang to disclose previous medical consultation and treatment constituted
died because of a kidney disorder. 4 A deposition given by Dr. Wilfredo Claudio was material information which should have been communicated to Great Pacific to
presented by petitioner. There Dr. Claudio stated that he was the family physician of enable the latter to make proper inquiries. The Court of Appeals finally held that
the deceased Jaime Canilang 5 and that he had previously treated him for "sinus the Ng Gan Zee case which had involved misrepresentation was not applicable in
tachycardia" and "acute bronchitis." 6 Great Pacific for its part presented Dr. respect of the case at bar which involves concealment.
Esperanza Quismorio, a physician
and a medical underwriter working for Great Pacific. 7 She testified that the Petitioner Thelma Canilang is now before this Court on a Petition for Review
on Certiorari alleging that:
29

1. . . . the Honorable Court of Appeals, speaking with due respect, erred in premium is paid and the Policy/TA/Certificate is delivered to, and accepted by me in
not holding that the issue in the case agreed upon between the parties person, when I am in actual good health.
before the Insurance Commission is whether or not Jaime Canilang
"intentionally" made material concealment in stating his state of health; Signed at Manila his 4th day of August, 1992.

2. . . . at any rate, the non-disclosure of certain facts about his previous We note that in addition to the negative statements made by Mr. Canilang in
health conditions does not amount to fraud and private respondent is paragraph 1 and 2 of the medical declaration, he failed to disclose in the
deemed to have waived inquiry thereto. 11 appropriate space, under the caption "Exceptions," that he had twice consulted Dr.
Wilfredo B. Claudio who had found him to be suffering from "sinus tachycardia" and
The medical declaration which was set out in the application for insurance executed "acute bronchitis."
by Jaime Canilang read as follows:
The relevant statutory provisions as they stood at the time Great Pacific issued the
MEDICAL DECLARATION contract of insurance and at the time Jaime Canilang died, are set out in P.D. No.
1460, also known as the Insurance Code of 1978, which went into effect on 11 June
I hereby declare that: 1978. These provisions read as follows:

(1) I have not been confined in any hospital, sanitarium or infirmary, nor receive any Sec. 26. A neglect to communicate that which a party
medical or surgical advice/attention within the last five (5) years. knows and ought to communicate, is called a concealment.

(2) I have never been treated nor consulted a physician for a heart condition, high Sec. 28. Each party to a contract of insurance must communicate
blood pressure, cancer, diabetes, lung, kidney, stomach disorder, or any other to the other, in good faith, all factorswithin his knowledge which
physical impairment. are material to the contract and as to which he makes no
warranty, and which the other has not the means of ascertaining.
(3) I am, to the best of my knowledge, in good health. (Emphasis supplied)

EXCEPTIONS: Under the foregoing provisions, the information concealed must be information
which the concealing party knew and "ought to [have] communicate[d]," that is to
say, information which was "material to the contract." The test of materiality is
____________________________________________________________________
contained in Section 31 of the Insurance Code of 1978 which reads:
____________

Sec. 31. Materially is to be determined not by the event,


GENERAL DECLARATION
but solely by the probable and reasonable influence of the facts
upon the party to whom the communication is due, in forming his
I hereby declare that all the foregoing answers and statements are complete, true
estimate of the disadvantages of the proposed contract, or in
and correct. I herebyagree that if there be any fraud or misrepresentation in the
making his inquiries. (Emphasis supplied)
above statements material to the risk, the INSURANCE COMPANY upon discovery
within two (2) years from the effective date of insurance shall have the right to
"Sinus tachycardia" is considered present "when the heart rate exceeds 100 beats
declare such insurance null and void. That the liabilities of the Company under the
per minute." 13 The symptoms of this condition include pounding in the chest and
said Policy/TA/Certificate shall accrue and begin only from the date of
sometimes faintness and weakness of the person affected. The following
commencement of risk stated in the Policy/TA/Certificate, provided that the first
30

elaboration was offered by Great Pacific and set out by the Court of Appeals in its information required of the applicant concerning previous
Decision: condition of health and diseases suffered, for such information
necessarily constitutes an important factor which the insurer takes
Sinus tachycardia is defined as sinus-initiated; heart rate faster than 100 into consideration in deciding whether to issue the policy or not . .
beats per minute. (Harrison' s Principles of Internal Medicine, 8th ed. . . 17 (Emphasis supplied)
[1978], p. 1193.) It is, among others, a common reaction to heart disease,
including myocardial infarction, and heart failure per se. (Henry J.L. The Insurance Commissioner had also ruled that the failure of Great Pacific to
Marriot, M.D.,Electrocardiography, 6th ed., [1977], p. 127.) The medication convey certain information to the insurer was not "intentional" in nature, for the
prescribed by Dr. Claudio for treatment of Canilang's ailment on June 18, reason that Jaime Canilang believed that he was suffering from minor ailment like a
1982, indicates the condition that said physician was trying to manage. common cold. Section 27 of the Insurance Code of 1978 as it existed from 1974 up to
Thus, he prescribed Trazepam, (Philippine Index of Medical 1985, that is, throughout the time range material for present purposes, provided
Specialties (PIMS), Vol. 14, No. 3, Dec. 1985, p. 112) which is anti-anxiety, that:
anti-convulsant, muscle-relaxant; and Aptin, (Idem, p. 36) a cardiac drug,
for palpitations and nervous heart. Such treatment could have been a very Sec. 27. A concealment entitles the injured party to rescind a
material information to the insurer in determining the action to be take on contract of insurance.
Canilang's application for life insurance coverage. 14
The preceding statute, Act No. 2427, as it stood from 1914 up to 1974, had
We agree with the Court of Appeals that the information which Jaime Canilang provided:
failed to disclose was material to the ability of Great Pacific to estimate the
probable risk he presented as a subject of life insurance. Had Canilang disclosed his Sec. 26. A concealment, whether intentional or unintentional,
visits to his doctor, the diagnosis made and medicines prescribed by such doctor, in entitles the injured party to rescind a contract of insurance.
the insurance application, it may be reasonably assumed that Great Pacific would (Emphasis supplied)
have made further inquiries and would have probably refused to issue a non-
medical insurance policy or, at the very least, required a higher premium for the
Upon the other hand, in 1985, the Insurance Code of 1978 was amended by
same coverage. 15 The materiality of the information withheld by Great Pacific did
B.P. Blg. 874. This subsequent statute modified Section 27 of the Insurance Code of
not depend upon the state of mind of Jaime Canilang. A man's state of mind or
1978 so as to read as follows:
subjective belief is not capable of proof in our judicial process, except through proof
of external acts or failure to act from which inferences as to his subjective belief may
Sec. 27. A concealment whether intentional or
be reasonably drawn. Neither does materiality depend upon the actual or physical
unintentional entitles the injured party to rescind a contract of
events which ensue. Materiality relates rather to the "probable and reasonable
insurance. (Emphasis supplied)
influence of the facts" upon the party to whom the communication should have
been made, in assessing the risk involved in making or omitting to make further
inquiries and in accepting the application for insurance; that "probable and The unspoken theory of the Insurance Commissioner appears to have been that by
reasonable influence of the facts" concealed must, of course, be determined deleting the phrase "intentional or unintentional," the Insurance Code of 1978 (prior
objectively, by the judge ultimately. to its amendment by B.P. Blg. 874) intended to limit the kinds of concealment which
generate a right to rescind on the part of the injured party to "intentional
concealments." This argument is not persuasive. As a simple matter of grammar, it
The insurance Great Pacific applied for was a "non-medical" insurance policy.
may be noted that "intentional" and "unintentional" cancel each other out. The net
In Saturnino v. Philippine-American Life Insurance Company, 16 this Court held that:
result therefore of the phrase "whether intentional or unitentional" is precisely to
leave unqualified the term "concealment." Thus, Section 27 of the Insurance Code of
. . . if anything, the waiver of medical examination [in a non-
1978 is properly read as referring to "any concealment" without regard to whether
medical insurance contract] renders even more material the
such concealment is intentional or unintentional. The phrase "whether intentional or
31

unintentional" was in fact superfluous. The deletion of the phrase "whether


intentional or unintentional" could not have had the effect of imposing an
affirmative requirement that a concealment must be intentional if it is to entitle the
injured party to rescind a contract of insurance. The restoration in 1985 by B.P. Blg.
874 of the phrase "whether intentional or unintentional" merely underscored the
fact that all throughout (from 1914 to 1985), the statute did not require proof that
concealment must be "intentional" in order to authorize rescission by the injured
party.

In any case, in the case at bar, the nature of the facts not conveyed to the insurer
was such that the failure to communicate must have been intentional rather than
merely inadvertent. For Jaime Canilang could not have been unaware that his heart
beat would at times rise to high and alarming levels and that he had consulted a
doctor twice in the two (2) months before applying for non-medical insurance.
Indeed, the last medical consultation took place just the day before the insurance
application was filed. In all probability, Jaime Canilang went to visit his doctor
precisely because of the discomfort and concern brought about by his experiencing
"sinus tachycardia."

We find it difficult to take seriously the argument that Great Pacific had waived
inquiry into the concealment by issuing the insurance policy notwithstanding
Canilang's failure to set out answers to some of the questions in the insurance
application. Such failure precisely constituted concealment on the part of Canilang.
Petitioner's argument, if accepted, would obviously erase Section 27 from the
Insurance Code of 1978.

It remains only to note that the Court of Appeals finding that the parties
had not agreed in the pretrial before the Insurance Commission that the relevant
issue was whether or not Jaime Canilang had intentionally concealed material
information from the insurer, was supported by the evidence of record, i.e., the Pre-
trial Order itself dated 17 October 1984 and the Minutes of the Pre-trial Conference
dated 15 October 1984, which "readily shows that the word "intentional" does not
appear in the statement or definition of the issue in the said Order and Minutes." 18

WHEREFORE, the Petition for Review is DENIED for lack of merit and the Decision of
the Court of Appeals dated 16 October 1989 in C.A.-G.R. SP No. 08696 is hereby
AFFIRMED. No pronouncement as to the costs.

SO ORDERED.

Vda. De Canilang v. CA - Concealment


32

223 SCRA 443 (1993) The information which Canilang failed to disclose was material to the ability of
Grepalife to estimate the probable risk he presented as a subject of life insurance.
Facts: Had Canilang disclosed his visits to his doctor, the diagnosis made and the
> Canilang consulted Dr. Claudio and was diagnosed as suffering from "sinus medicines prescribed by such doctor, in the insurance application, it may be
tachycardia." Mr. Canilang consulted the same doctor again on 3 August 1982 and reasonably assumed that Grepalife would have made further inquiries and would
this time was found to have "acute bronchitis." have probably refused to issue a non-medical insurance policy or, at the very least,
required a higher premium for the same coverage.
> On the next day, 4 August 1982, Canilang applied for a "non-medical" insurance
policy with Grepalife naming his wife, as his beneficiary. Canilang was issued
ordinary life insurance with the face value of P19,700. The materiality of the information withheld by Canilang from Grepalife did not
> On 5 August 1983, Canilang died of "congestive heart failure," "anemia," and depend upon the state of mind of Jaime Canilang. A man's state of mind or
"chronic anemia." The wife as beneficiary, filed a claim with Grepalife which the subjective belief is not capable of proof in our judicial process, except through proof
insurer denied on the ground that the insured had concealed material information of external acts or failure to act from which inferences as to his subjective belief
from it. may be reasonably drawn. Neither does materiality depend upon the actual or
physical events which ensue. Materiality relates rather to the "probable and
> Vda Canilang filed a complaint with the Insurance Commissioner against Grepalife reasonable influence of the facts" upon the party to whom the communication
contending that as far as she knows her husband was not suffering from any should have been made, in assessing the risk involved in making or omitting to
disorder and that he died of kidney disorder. make further inquiries and in accepting the application for insurance; that
> Grepalife was ordered to pay the widow by the Insurance Commissioner holding "probable and reasonable influence of the facts" concealed must, of course, be
that there was no intentional concealment on the Part of Canilang and that determined objectively, by the judge ultimately.
Grepalife had waived its right to inquire into the health condition of the applicant
by the issuance of the policy despite the lack of answers to "some of the pertinent
questions" in the insurance application. CA reversed. SC found it difficult to take seriously the argument that Grepalife had waived
inquiry into the concealment by issuing the insurance policy notwithstanding
Issue: Canilang's failure to set out answers to some of the questions in the insurance
application. Such failure precisely constituted concealment on the part of Canilang.
Whether or not Grepalife is liable. Petitioner's argument, if accepted, would obviously erase Section 27 from the
Insurance Code of 1978.
Held:
Facts:
SC took note of the fact that Canilang failed to disclose that hat he had twice Canilang was found to have suffered from sinus tachycardia then bronchitis after a
consulted Dr. Wilfredo B. Claudio who had found him to be suffering from "sinus check-up from his doctor. The next day, he applied for a "non-medical" insurance
tachycardia" and "acute bronchitis. Under the relevant provisions of the Insurance policy with respondent Grepalife naming his wife, Thelma Canilang, as his
Code, the information concealed must be information which the concealing party beneficiary. This was to the value of P19,700.
knew and "ought to [have] communicate[d]," that is to say, information which was He died of "congestive heart failure," "anemia," and "chronic anemia." The widow
"material to the contract. filed a claim with Great Pacific which the insurer denied on the ground that the
insured had concealed material information from it.
33

Petitioner then filed a complaint against Great Pacific for recovery of the insurance The information concealed must be information which the concealing party knew
proceeds. Petitioner testified that she was not aware of any serious illness suffered and should have communicated. The test of materiality of such information is
by her late husband and her husband had died because of a kidney disorder. The contained in Section 31:
doctor who gave the check up stated that he treated the deceased for “sinus Sec. 31. Materiality is to be determined not by the event, but solely by the probable
tachycardia” and "acute bronchitis." and reasonable influence of the facts upon the party to whom the communication
Great Pacific presented a physician who testified that the deceased's insurance is due, in forming his estimate of the disadvantages of the proposed contract, or in
application had been approved on the basis of his medical declaration. She making his inquiries.
explained that as a rule, medical examinations are required only in cases where the The information which Jaime Canilang failed to disclose was material to the ability
applicant has indicated in his application for insurance coverage that he has of Great Pacific to estimate the probable risk he presented as a subject of life
previously undergone medical consultation and hospitalization. insurance. Had he disclosed his visits to his doctor, the diagnosis made and
The Insurance Commissioner ordered Great Pacific to pay P19,700 plus legal medicines prescribed by such doctor, in the insurance application, it may be
interest and P2,000.00 as attorney's fees. On appeal by Great Pacific, the Court of reasonably assumed that Great Pacific would have made further inquiries and
Appeals reversed. It found that the failure of Jaime Canilang to disclose previous would have probably refused to issue a non-medical insurance policy.
medical consultation and treatment constituted material information which should Materiality relates rather to the "probable and reasonable influence of the facts"
have been communicated to Great Pacific to enable the latter to make proper upon the party to whom the communication should have been made, in assessing
inquiries. the risk involved in making or omitting to make further inquiries and in accepting
Hence this petition by the widow. the application for insurance; that "probable and reasonable influence of the facts"
concealed must, of course, be determined objectively, by the judge ultimately.
Issue: Won Canilang was guilty of misrepresentation The Insurance Commissioner had also ruled that the failure of Great Pacific to
convey certain information to the insurer was not "intentional" in nature, for the
Held: Yes. Petition denied. reason that Canilang believed that he was suffering from minor ailment like a
common cold. Section 27 stated that:
Ratio: Sec. 27. A concealment whether intentional or unintentional entitles the injured
There was a right of the insurance company to rescind the contract if it was proven party to rescind a contract of insurance.
that the insured committed fraud in not affirming that he was treated for heart The failure to communicate must have been intentional rather than inadvertent.
condition and other ailments stipulated. Canilang could not have been unaware that his heart beat would at times rise to
Apart from certifying that he didn’t suffer from such a condition, Canilang also high and alarming levels and that he had consulted a doctor twice in the two (2)
failed to disclose in the that he had twice consulted a doctor who had found him to months before applying for non-medical insurance. Indeed, the last medical
be suffering from "sinus tachycardia" and "acute bronchitis." consultation took place just the day before the insurance application was filed. In all
Under the Insurance Code: probability, Jaime Canilang went to visit his doctor precisely because of the ailment.
Sec. 26. A neglect to communicate that which a party knows and ought to Canilang's failure to set out answers to some of the questions in the insurance
communicate, is called a concealment. application constituted concealment
Sec. 28. Each party to a contract of insurance must communicate to the other, in
good faith, all factors within his knowledge which are material to the contract and
as to which he makes no warranty, and which the other has not the means of THELMA vda. de CANILANG v. CA and GREAT PACIFIC LIFE ASSURANCE
ascertaining. 1993 / Feliciano / Petition for review on certiorari of CA decision
On June 1982, Jaime Canilang was diagnosed as suffering from sinus tachycardia.
Two months later, he was found to have acute bronchitis. The next day, he applied
for a “non-medical” insurance policy with Great Pacific and named his wife Thelma
34

as his beneficiary. A year later, he died of congestive heart failure, anemia, and
chronic anemia. When Thelma filed a claim with Great Pacific, it was denied on the
ground that Jaime concealed material information.
Thelma filed a complaint against Great Pacific with the Insurance
Commission for recovery of the insurance proceeds. She testified that she was not
aware of any serious illness suffered by Jaime, and that what she knew was that he
died because of a kidney disorder. Great Pacific presented a physician who
explained that Jaime’s application had been approved based on his medical
declaration, and that medical examinations are required only in cases where
applicant indicated that he has undergone medical consultation and hospitalization.
The Insurance Commission held that there was no intentional
concealment on Jaime’s part. It also held that Great Pacific waived its right to
inquire into Jaime’s health condition by issuing the policy despite the lack of
answers to some of the pertinent questions in the application. It said BP 874, which
voids an insurance contract WON concealment was made intentionally, was not
applicable since the law became effective only on 1985.
CA reversed IC. CA said that the issue is WON there was material
concealment, and not WON Canilang ‘intentionally’ made material concealment. It
held that Jaime’s failure to disclose previous medical consultation and treatment
constituted material information.
CANILANG FAILED TO DISCLOSE MATERIAL INFORMATION
The applicable law at that time was PD 1460 (Insurance Code of 1978). Under said
law, the information concealed must be such which the concealing party knew and
ought to have communicated—those which are material to the contract. The test of
materiality is determined not by the event, but solely by the probable and
reasonable influence of the facts upon the party to whom communication is due,
in forming his estimate of the disadvantages of the proposed contract, or in making
his inquiries.
Canilang failed to disclose material information when he did not indicate
under the caption ‘Exceptions’ that he twice consulted a doctor who found him to
be suffering from sinus tachycardia and acute bronchitis. This failure to
communicate must have been intentional, since Jaime could have been aware that
his heartbeat would rise to high levels and that he consulted a doctor twice before
applying for insurance.
The preceding statute, Act 2427, provided that a concealment, whether
intentional or unintentional, entitles the injured party to rescind a contract of
insurance. However, in PD 1460, this phrase was not present. [The current law, BP
874, has the phrase.] SC rejected the IC’s unspoken theory that the deletion of the
phrase intended to limit the kinds of concealment to intentionalconcealments. The
provision is properly read as referring to ANY concealment [“intentional” and
“unintentional” cancel each other out].
CA AFFIRMED; PETITION DENIED
35

FIRST DIVISION a) consulted any doctor or other health practitioner?

G.R. No. 105135 June 22, 1995 b) submitted to:

SUNLIFE ASSURANCE COMPANY OF CANADA, petitioner, EGG?


vs. X-rays?
The Hon. COURT OF APPEALS and Spouses ROLANDO and BERNARDA blood tests?
BACANI, respondents. other tests?

QUIASON, J.: c) attended or been admitted to any hospital or other medical facility?

This is a petition for review for certiorari under Rule 45 of the Revised Rules of 6. Have you ever had or sought advice for:
Court to reverse and set aside the Decision dated February 21, 1992 of the Court of
Appeals in CA-G.R. CV No. 29068, and its Resolution dated April 22, 1992, denying b) urine, kidney or bladder disorder? (Rollo, p. 53)
reconsideration thereof.
The deceased answered question No. 5(a) in the affirmative but limited his answer
We grant the petition. to a consultation with a certain Dr. Reinaldo D. Raymundo of the Chinese General
Hospital on February 1986, for cough and flu complications. The other questions
I were answered in the negative (Rollo, p. 53).

On April 15, 1986, Robert John B. Bacani procured a life insurance contract for Petitioner discovered that two weeks prior to his application for insurance, the
himself from petitioner. He was issued Policy No. 3-903-766-X valued at insured was examined and confined at the Lung Center of the Philippines, where he
P100,000.00, with double indemnity in case of accidental death. The designated was diagnosed for renal failure. During his confinement, the deceased was
beneficiary was his mother, respondent Bernarda Bacani. subjected to urinalysis, ultra-sonography and hematology tests.

On June 26, 1987, the insured died in a plane crash. Respondent Bernarda Bacani On November 17, 1988, respondent Bernarda Bacani and her husband, respondent
filed a claim with petitioner, seeking the benefits of the insurance policy taken by Rolando Bacani, filed an action for specific performance against petitioner with the
her son. Petitioner conducted an investigation and its findings prompted it to reject Regional Trial Court, Branch 191, Valenzuela, Metro Manila. Petitioner filed its
the claim. answer with counterclaim and a list of exhibits consisting of medical records
furnished by the Lung Center of the Philippines.
In its letter, petitioner informed respondent Bernarda Bacani, that the insured did
not disclose material facts relevant to the issuance of the policy, thus rendering the On January 14, 1990, private respondents filed a "Proposed Stipulation with Prayer
contract of insurance voidable. A check representing the total premiums paid in the for Summary Judgment" where they manifested that they "have no evidence to
amount of P10,172.00 was attached to said letter. refute the documentary evidence of concealment/misrepresentation by the
decedent of his health condition (Rollo, p. 62).
Petitioner claimed that the insured gave false statements in his application when he
answered the following questions: Petitioner filed its Request for Admissions relative to the authenticity and due
execution of several documents as well as allegations regarding the health of the
5. Within the past 5 years have you: insured. Private respondents failed to oppose said request or reply thereto, thereby
rendering an admission of the matters alleged.
36

Petitioner then moved for a summary judgment and the trial court decided in favor In weighing the evidence presented, the trial court concluded that indeed there was
of private respondents. The dispositive portion of the decision is reproduced as concealment and misrepresentation, however, the same was made in "good faith"
follows: and the facts concealed or misrepresented were irrelevant since the policy was
"non-medical". We disagree.
WHEREFORE, judgment is hereby rendered in favor of the plaintiffs and
against the defendant, condemning the latter to pay the former the Section 26 of The Insurance Code is explicit in requiring a party to a contract of
amount of One Hundred Thousand Pesos (P100,000.00) the face value of insurance to communicate to the other, in good faith, all facts within his knowledge
insured's Insurance Policy No. 3903766, and the Accidental Death Benefit which are material to the contract and as to which he makes no warranty, and
in the amount of One Hundred Thousand Pesos (P100,000.00) and further which the other has no means of ascertaining. Said Section provides:
sum of P5,000.00 in the concept of reasonable attorney's fees and costs of
suit. A neglect to communicate that which a party knows and ought to
communicate, is called concealment.
Defendant's counterclaim is hereby Dismissed (Rollo, pp. 43-44).
Materiality is to be determined not by the event, but solely by the probable and
In ruling for private respondents, the trial court concluded that the facts concealed reasonable influence of the facts upon the party to whom communication is due, in
by the insured were made in good faith and under a belief that they need not be forming his estimate of the disadvantages of the proposed contract or in making his
disclosed. Moreover, it held that the health history of the insured was immaterial inquiries (The Insurance Code, Sec. 31).
since the insurance policy was "non-medical".
The terms of the contract are clear. The insured is specifically required to disclose
Petitioner appealed to the Court of Appeals, which affirmed the decision of the trial to the insurer matters relating to his health.
court. The appellate court ruled that petitioner cannot avoid its obligation by
claiming concealment because the cause of death was unrelated to the facts The information which the insured failed to disclose were material and relevant to
concealed by the insured. It also sustained the finding of the trial court that matters the approval and issuance of the insurance policy. The matters concealed would
relating to the health history of the insured were irrelevant since petitioner waived have definitely affected petitioner's action on his application, either by approving it
the medical examination prior to the approval and issuance of the insurance policy. with the corresponding adjustment for a higher premium or rejecting the same.
Moreover, the appellate court agreed with the trial court that the policy was "non- Moreover, a disclosure may have warranted a medical examination of the insured
medical" (Rollo, pp. 4-5). by petitioner in order for it to reasonably assess the risk involved in accepting the
application.
Petitioner's motion for reconsideration was denied; hence, this petition.
In Vda. de Canilang v. Court of Appeals, 223 SCRA 443 (1993), we held that
II materiality of the information withheld does not depend on the state of mind of the
insured. Neither does it depend on the actual or physical events which ensue.
We reverse the decision of the Court of Appeals.
Thus, "goad faith" is no defense in concealment. The insured's failure to disclose the
The rule that factual findings of the lower court and the appellate court are binding fact that he was hospitalized for two weeks prior to filing his application for
on this Court is not absolute and admits of exceptions, such as when the judgment insurance, raises grave doubts about his bonafides. It appears that such
is based on a misappreciation of the facts (Geronimo v. Court of Appeals, 224 SCRA concealment was deliberate on his part.
494 [1993]).
The argument, that petitioner's waiver of the medical examination of the insured
debunks the materiality of the facts concealed, is untenable. We reiterate our ruling
37

in Saturnino v. Philippine American Life Insurance Company, 7 SCRA 316 (1963), that
" . . . the waiver of a medical examination [in a non-medical insurance contract]
renders even more material the information required of the applicant concerning
previous condition of health and diseases suffered, for such information necessarily
constitutes an important factor which the insurer takes into consideration in
deciding whether to issue the policy or not . . . "

Moreover, such argument of private respondents would make Section 27 of the


Insurance Code, which allows the injured party to rescind a contract of insurance
where there is concealment, ineffective (See Vda. de Canilang v. Court of
Appeals, supra).

Anent the finding that the facts concealed had no bearing to the cause of death of
the insured, it is well settled that the insured need not die of the disease he had
failed to disclose to the insurer. It is sufficient that his non-disclosure misled the
insurer in forming his estimates of the risks of the proposed insurance policy or in
making inquiries (Henson v. The Philippine American Life Insurance Co., 56 O.G. No.
48 [1960]).

We, therefore, rule that petitioner properly exercised its right to rescind the
contract of insurance by reason of the concealment employed by the insured. It
must be emphasized that rescission was exercised within the two-year
contestability period as recognized in Section 48 of The Insurance Code.

WHEREFORE, the petition is GRANTED and the Decision of the Court of Appeals is
REVERSED and SET ASIDE.

SO ORDERED.
38

245 SCRA 268 (1995) o ISSUE:

o INSURANCE LAW: Concealment o Whether or not the concealment made by Bacani warranted the rejection of the
o Disclosure of material facts is required insurance claim
o Good faith is not a defense in determining the materiality of the information to be
disclosed HELD:
o Waiver of medical examination by insured is not a defense
The Supreme Court reversed the decision of the CA and ruled that rescission of the
o Cause of death is immaterial in case of concealment
insurance contract was proper.

FACTS: Disclosure of Material Facts required


o
Bacani procured a life insurance contract for himself from Sunlife Assurance. Under sec. 26 of the Insurance Code, a party to a contract of insurance is required
Specifically, the policy included a double indemnity in case of accidental death, to communicate to the other, in good faith, all facts within his knowledge which are
material to the contract and as to which he makes no warranty, and which the
designating his mother as beneficiary.
other has no means of ascertaining.

Later, Bacani died in a plane crash and so the mother filed a claim. After Materiality is to be determined not by the event, but solely by the probable and
investigation, Sunlife rejected the claim on ground of non-disclosure of material reasonable influence of the facts upon the party to whom communication is due, in
facts. They said that Bacani did not mention that two weeks prior to his insurance forming his estimate of the disadvantages of the proposed contract or in making his
application he was examined and confined at the Lung Center of the Philippines, inquiries. (The Insurance Code, sec. 31)
where he was diagnosed for renal failure.
The information which the insured failed to disclose was material and relevant to
the approval and issuance of the insurance policy. The matters concealed would
The trial court ruled that the facts concealed by the insured were made in good have definitely affected petitioner’s action on his application, either by approving it
faith and under the belief that they need not be disclosed. Also, it held that the with the corresponding adjustment for a higher premium or rejecting the same.
health history of the insured was immaterial since the insurance policy was “non- Moreover, a disclosure may have warranted a medical examination of the insured
medical.” by the petitioner in order for it to reasonably assess the risk involved in accepting
the application.

The CA affirmed, stating that the cause of death was unrelated to the facts
Good Faith not a defense
concealed by the insured.
Materiality of the information withheld does not depend on the state of mind of
the insured. Neither does it depend on the actual or physical events which ensue.

Thus, “good faith” is no defense in concealment.


39

Waiver of Medical Examination not a defense

The waiver of the medical examination of the insured does not mean that material
facts need not be disclosed. In fact, it renders even more material the information
required of the applicant concerning previous condition of health and diseases
suffered, for such information necessarily constitutes an important factor which the
insurer takes into consideration in deciding whether to issue the policy or not.

Cause of Death

It is well settled that the insured need not die of the disease he had failed to
disclose to the insurer. It is sufficient that his non-disclosure misled the insurer in
forming his estimates of the risks of the proposed insurance policy or in making
inquiries.
40

THIRD DIVISION Plans.[5] Under the master policy, Philam Life was to automatically provide life
MA. LOURDES S. FLORENDO, G.R. No. 186983
insurance coverage, including accidental death, to all who signed up for Philam
Petitioner, Plans comprehensive pension plan.[6] If the plan holder died before the maturity of
Present: the plan, his beneficiary was to instead receive the proceeds of the life insurance,
VELASCO, JR., J., Chairperson,
equivalent to the pre-need price. Further, the life insurance was to take care of any
- versus - PERALTA,
unpaid premium until the pension plan matured, entitling the beneficiary to the
ABAD MENDOZA, and PERLAS-BERNABE, JJ. maturity value of the pension plan.[7]

PHILAM PLANS, INC.,


On October 30, 1997 Philam Plans issued Pension Plan Agreement PP43005584[8] to
PERLA ABCEDE and Promulgated: Manuel, with petitioner Ma. Lourdes S. Florendo, his wife, as beneficiary. In time,
Manuel paid his quarterly premiums.[9]
MA. CELESTE ABCEDE,
Eleven months later or on September 15, 1998, Manuel died of blood
Respondents. February 22, 2012
poisoning. Subsequently, Lourdes filed a claim with Philam Plans for the payment of
DECISION the benefits under her husbands plan.[10] Because Manuel died before his pension
ABAD, J.: plan matured and his wife was to get only the benefits of his life insurance, Philam
This case is about an insureds alleged concealment in his pension plan application Plans forwarded her claim to Philam Life.[11]
of his true state of health and its effect on the life insurance portion of that plan in
case of death. On May 3, 1999 Philam Plans wrote Lourdes a letter,[12] declining her claim. Philam
Life found that Manuel was on maintenance medicine for his heart and had an
The Facts and the Case implanted pacemaker. Further, he suffered from diabetes mellitus and was taking
insulin. Lourdes renewed her demand for payment under the plan[13] but Philam
On October 23, 1997 Manuel Florendo filed an application for comprehensive
Plans rejected it,[14] prompting her to file the present action against the pension
pension plan with respondent Philam Plans, Inc. (Philam Plans) after some
plan company before the Regional Trial Court (RTC) of Quezon City.[15]
convincing by respondent Perla Abcede. The plan had a pre-need price
of P997,050.00, payable in 10 years, and had a maturity value of P2,890,000.00 On March 30, 2006 the RTC rendered judgment,[16] ordering Philam Plans, Perla and
after 20 years.[1] Manuel signed the application and left to Perla the task of Ma. Celeste, solidarily, to pay Lourdes all the benefits from her husbands pension
supplying the information needed in the application.[2] Respondent Ma. Celeste plan, namely: P997,050.00, the proceeds of his term insurance, and P2,890,000.00
Abcede, Perlas daughter, signed the application as sales counselor.[3] lump sum pension benefit upon maturity of his plan; P100,000.00 as moral
damages; and to pay the costs of the suit. The RTC ruled that Manuel was not guilty
Aside from pension benefits, the comprehensive pension plan also provided life
of concealing the state of his health from his pension plan application.
insurance coverage to Florendo.[4] This was covered by a Group Master Policy that
Philippine American Life Insurance Company (Philam Life) issued to Philam
41

On December 18, 2007 the Court of Appeals (CA) reversed the RTC anyone that he had been under treatment for heart condition and diabetes for
[17]
decision, holding that insurance policies are traditionally contracts uberrimae more than five years preceding his submission of that application. But he kept those
fidae or contracts of utmost good faith. As such, it required Manuel to disclose to crucial facts from Philam Plans.
Philam Plans conditions affecting the risk of which he was aware or material facts
Besides, when Manuel signed the pension plan application, he adopted as his own
that he knew or ought to know.[18]
the written representations and declarations embodied in it. It is clear from these
Issues Presented representations that he concealed his chronic heart ailment and diabetes from
Philam Plans. The pertinent portion of his representations and declarations read as
The issues presented in this case are:
follows:

1. Whether or not the CA erred in finding Manuel guilty of concealing his illness I hereby represent and declare to the best of my knowledge that:
when he kept blank and did not answer questions in his pension plan application
(c) I have never been treated for heart condition, high blood
regarding the ailments he suffered from;
pressure, cancer, diabetes, lung, kidney or stomach
2. Whether or not the CA erred in holding that Manuel was bound by the failure of disorder or any other physical impairment in the last five
respondents Perla and Ma. Celeste to declare the condition of Manuels health in years.
the pension plan application; and
(d) I am in good health and physical condition.
3. Whether or not the CA erred in finding that Philam Plans approval of Manuels
pension plan application and acceptance of his premium payments precluded it If your answer to any of the statements above reveal
from denying Lourdes claim. otherwise, please give details in the space provided for:

Date of confinement : ____________________________


Rulings of the Court
Name of Hospital or Clinic :
One. Lourdes points out that, seeing the unfilled spaces in Manuels pension plan ____________________________
application relating to his medical history, Philam Plans should have returned it to
Name of Attending Physician :
him for completion. Since Philam Plans chose to approve the application just as it
____________________________
was, it cannot cry concealment on Manuels part. Further, Lourdes adds that Philam
Plans never queried Manuel directly regarding the state of his health. Consequently, Findings : ____________________________
it could not blame him for not mentioning it.[19]
Others: (Please specify) :
But Lourdes is shifting to Philam Plans the burden of putting on the pension plan ____________________________

application the true state of Manuels health. She forgets that since Philam Plans x x x x.[20] (Emphasis supplied)
waived medical examination for Manuel, it had to rely largely on his stating the
truth regarding his health in his application. For, after all, he knew more than
42

Since Manuel signed the application without filling in the details regarding his claimed that Perla was aware of his two other afflictions that needed medical
continuing treatments for heart condition and diabetes, the assumption is that he treatments. Pursuant to Section 27[27] of the Insurance Code, Manuels concealment
has never been treated for the said illnesses in the last five years preceding his entitles Philam Plans to rescind its contract of insurance with him.
application. This is implicit from the phrase If your answer to any of the statements
Two. Lourdes contends that the mere fact that Manuel signed the application in
above (specifically, the statement: I have never been treated for heart condition or
blank and let Perla fill in the required details did not make her his agent and bind
diabetes) reveal otherwise, please give details in the space provided for. But this is
him to her concealment of his true state of health. Since there is no evidence of
untrue since he had been on Coumadin, a treatment for venous thrombosis, [21] and
collusion between them, Perlas fault must be considered solely her own and cannot
insulin, a drug used in the treatment of diabetes mellitus, at that time.[22]
prejudice Manuel.[28]
Lourdes insists that Manuel had concealed nothing since Perla, the soliciting agent,
But Manuel forgot that in signing the pension plan application, he certified that he
knew that Manuel had a pacemaker implanted on his chest in the 70s or about 20
wrote all the information stated in it or had someone do it under his
years before he signed up for the pension plan.[23] But by its tenor, the responsibility
direction. Thus:
for preparing the application belonged to Manuel. Nothing in it implies that
someone else may provide the information that Philam Plans needed. Manuel APPLICATION FOR PENSION PLAN
cannot sign the application and disown the responsibility for having it filled up. If he
(Comprehensive)
furnished Perla the needed information and delegated to her the filling up of the
application, then she acted on his instruction, not on Philam Plans instruction. I hereby apply to purchase from PHILAM PLANS, INC. a Pension
Plan Program described herein in accordance with the General
Lourdes next points out that it made no difference if Manuel failed to reveal the Provisions set forth in this application and hereby certify that the
fact that he had a pacemaker implant in the early 70s since this did not fall within date and other information stated herein are written by me or
the five-year timeframe that the disclosure contemplated. [24] But a pacemaker is an under my direction. x x x.[29] (Emphasis supplied)
electronic device implanted into the body and connected to the wall of the heart,
Assuming that it was Perla who filled up the application form, Manuel is still bound
designed to provide regular, mild, electric shock that stimulates the contraction of
by what it contains since he certified that he authorized her action. Philam Plans
the heart muscles and restores normalcy to the heartbeat. [25] That Manuel still had
had every right to act on the faith of that certification.
his pacemaker when he applied for a pension plan in October 1997 is an admission
that he remained under treatment for irregular heartbeat within five years Lourdes could not seek comfort from her claim that Perla had assured Manuel that
preceding that application. the state of his health would not hinder the approval of his application and that
what is written on his application made no difference to the insurance
Besides, as already stated, Manuel had been taking medicine for his heart condition
company. But, indubitably, Manuel was made aware when he signed the pension
and diabetes when he submitted his pension plan application. These clearly fell
plan application that, in granting the same, Philam Plans and Philam Life were
within the five-year period. More, even if Perlas knowledge of Manuels pacemaker
acting on the truth of the representations contained in that application. Thus:
may be applied to Philam Plans under the theory of imputed knowledge,[26] it is not
43

DECLARATIONS AND REPRESENTATIONS The Court cannot agree. The comprehensive pension plan that Philam Plans issued
contains a one-year incontestability period. It states:
I agree that the insurance coverage of this application is based
on the truth of the foregoing representations and is subject to
VIII. INCONTESTABILITY
the provisions of the Group Life Insurance Policy issued by THE
PHILIPPINE AMERICAN LIFE INSURANCE CO. to PHILAM PLANS, After this Agreement has remained in force for one (1) year, we
INC.[30] (Emphasis supplied) can no longer contest for health reasons any claim for insurance
under this Agreement, except for the reason that installment has
As the Court said in New Life Enterprises v. Court of Appeals:[31] not been paid (lapsed), or that you are not insurable at the time
you bought this pension program by reason of age. If this
It may be true that x x x insured persons may accept policies without Agreement lapses but is reinstated afterwards, the one (1) year
reading them, and that this is not negligence per se. But, this is not without contestability period shall start again on the date of approval of
any exception. It is and was incumbent upon petitioner Sy to read the your request for reinstatement.[35]
insurance contracts, and this can be reasonably expected of him
The above incontestability clause precludes the insurer from disowning
considering that he has been a businessman since 1965 and the contract
liability under the policy it issued on the ground of concealment or
concerns indemnity in case of loss in his money-making trade of which
misrepresentation regarding the health of the insured after a year of its
important consideration he could not have been unaware as it was
issuance.
precisely the reason for his procuring the same.[32]
Since Manuel died on the eleventh month following the issuance of his plan,[36] the
The same may be said of Manuel, a civil engineer and manager of a construction
one year incontestability period has not yet set in. Consequently, Philam Plans was
company.[33] He could be expected to know that one must read every document,
not barred from questioning Lourdes entitlement to the benefits of her husbands
especially if it creates rights and obligations affecting him, before signing the
pension plan.
same. Manuel is not unschooled that the Court must come to his succor. It could
reasonably be expected that he would not trifle with something that would provide WHEREFORE, the Court AFFIRMS in its entirety the decision of the Court of Appeals
additional financial security to him and to his wife in his twilight years. in CA-G.R. CV 87085 dated December 18, 2007.

Three. In a final attempt to defend her claim for benefits under Manuels pension SO ORDERED.
plan, Lourdes points out that any defect or insufficiency in the information provided
by his pension plan application should be deemed waived after the same has been
approved, the policy has been issued, and the premiums have been collected. [34]
44

G.R. No. 186983 February 22, 2012 incontestability clause


precludes the insurer from disowning liability under thepolicy it issued on the ground of concealment
MA. LOURDES S. FLORENDO, or misrepresentation regarding the health of theinsured after a year of its issuance.Since
Petitioner,vs. Manuel died on the eleventh month
PHILAM PLANS, INC., PERLA ABCEDE MA. CELESTE ABCEDE, following the issuance of his plan, the one year incontestability period has not yet set in.
Respondents. Consequently, Philam Plans was not barred from
questioning Lourdes’ entitlement to the benefits of her husband’s pension plan.
FACTS:

Manuel Florendo filed an application for comprehensive pension plan with respondent PhilamPlans,
Inc. (Philam Plans) Manuel signed the application and left to Perla the task of supplying
the information needed in the application. Respondent Ma. Celeste Abcede, Perla’s
daughter,
signed the application as sales counselor. Philam Plans issued Pension Plan Agreement toManuel,
with petitioner Ma. Lourdes S. Florendo, his wife, as beneficiary. In time, Manuel paidhis quarterly
premiums. Eleven months later, Manuel died of blood poisoning. Subsequently,Lourdes filed a claim
with Philam Plans for th
e payment of the benefits under her husband’s
plan but Philam Plans declined her claim prompting her to file the present action against thepension
plan company before the Regional Trial Court (RTC) of Quezon City and ruled in favor of Ma. Lourdes.
However, the Court of Appeals then reversed the RTC decision. Hence thisappeal.

ISSUE:

Whether or not Ma. Lourdes could claim benefits as the beneficiary of her husband under
theinsurance plan despite consideration that her husband Manuel concealed the true condition
of his health.

RULING:

The Supreme Court answers this to the negative and the AFFIRMED in its entirety the decisionof the
Court of Appeals.The comprehensive pension plan that Philam Plans issued contains a one-year
incontestabilityperiod. It states:

VIII. INCONTESTABILITY

After this Agreement has remained in force for one (1) year, we can no longer cont
est for health reasons any claim for insurance under this Agreement, except for the reason
thatinstallment has not been paid (lapsed), or that you are not insurable at the time you bought
thispension program by reason of age. If this Agreement lapses but is reinstated afterwards, theone
(1) year contestability period shall start again on the date of approval of your request
for reinstatement.The above
45

EN BANC Bulacan, as a result of which mishap Federico Songco (father) and Rodolfo Songco
(son) died, Carlos Songco (another son), the latter's wife, Angelita Songco, and a
G.R. No. L-24833 September 23, 1968 family friend by the name of Jose Manuel sustained physical injuries of varying
degree." 1
FIELDMEN'S INSURANCE CO., INC., petitioner,
vs. It was further shown according to the decision of respondent Court of Appeals:
MERCEDES VARGAS VDA. DE SONGCO, ET AL. and COURT OF "Amor Songco, 42-year-old son of deceased Federico Songco, testifying as witness,
APPEALS, respondents. declared that when insurance agent Benjamin Sambat was inducing his father to
insure his vehicle, he butted in saying: 'That cannot be, Mr. Sambat, because our
Jose S. Suarez for petitioner. vehicle is an "owner" private vehicle and not for passengers,' to which agent
Eligio G. Lagman for respondents. Sambat replied: 'whether our vehicle was an "owner" type or for passengers it
could be insured because their company is not owned by the Government and the
Government has nothing to do with their company. So they could do what they
FERNANDO, J.:
please whenever they believe a vehicle is insurable' ... In spite of the fact that the
present case was filed and tried in the CFI of Pampanga, the defendant company
An insurance firm, petitioner Fieldmen's Insurance Co., Inc., was not allowed to
did not even care to rebut Amor Songco's testimony by calling on the witness-stand
escape liability under a common carrier insurance policy on the pretext that what
agent Benjamin Sambat, its Pampanga Field Representative." 2
was insured, not once but twice, was a private vehicle and not a common carrier,
the policy being issued upon the insistence of its agent who discounted fears of the
The plaintiffs in the lower court, likewise respondents here, were the surviving
insured that his privately owned vehicle might not fall within its terms, the insured
widow and children of the deceased Federico Songco as well as the injured
moreover being "a man of scant education," finishing only the first grade. So it was
passenger Jose Manuel. On the above facts they prevailed, as had been mentioned,
held in a decision of the lower court thereafter affirmed by respondent Court of
in the lower court and in the respondent Court of Appeals.1awphîl.nèt
Appeals. Petitioner in seeking the review of the above decision of respondent Court
of Appeals cannot be so sanguine as to entertain the belief that a different outcome
could be expected. To be more explicit, we sustain the Court of Appeals. The basis for the favorable judgment is the doctrine announced in Qua Chee Gan v.
Law Union and Rock Insurance Co., Ltd., 3 with Justice J. B. L. Reyes speaking for the
Court. It is now beyond question that where inequitable conduct is shown by an
The facts as found by respondent Court of Appeals, binding upon us, follow: "This is
insurance firm, it is "estopped from enforcing forfeitures in its favor, in order to
a peculiar case. Federico Songco of Floridablanca, Pampanga, a man of scant
forestall fraud or imposition on the insured." 4
education being only a first grader ..., owned a private jeepney with Plate No. 41-
289 for the year 1960. On September 15, 1960, as such private vehicle owner, he
was induced by Fieldmen's Insurance Company Pampanga agent Benjamin Sambat As much, if not much more so than the Qua Chee Gan decision, this is a case where
to apply for a Common Carrier's Liability Insurance Policy covering his motor vehicle the doctrine of estoppel undeniably calls for application. After petitioner Fieldmen's
... Upon paying an annual premium of P16.50, defendant Fieldmen's Insurance Insurance Co., Inc. had led the insured Federico Songco to believe that he could
Company, Inc. issued on September 19, 1960, Common Carriers Accident Insurance qualify under the common carrier liability insurance policy, and to enter into
Policy No. 45-HO- 4254 ... the duration of which will be for one (1) year, effective contract of insurance paying the premiums due, it could not, thereafter, in any
September 15, 1960 to September 15, 1961. On September 22, 1961, the defendant litigation arising out of such representation, be permitted to change its stand to the
company, upon payment of the corresponding premium, renewed the policy by detriment of the heirs of the insured. As estoppel is primarily based on the doctrine
extending the coverage from October 15, 1961 to October 15, 1962. This time of good faith and the avoidance of harm that will befall the innocent party due to its
Federico Songco's private jeepney carried Plate No. J-68136-Pampanga-1961. ... On injurious reliance, the failure to apply it in this case would result in a gross travesty
October 29, 1961, during the effectivity of the renewed policy, the insured vehicle of justice.
while being driven by Rodolfo Songco, a duly licensed driver and son of Federico
(the vehicle owner) collided with a car in the municipality of Calumpit, province of
46

That is all that needs be said insofar as the first alleged error of respondent Court of weaker party may not change one whit, his participation in the 'agreement' being
Appeals is concerned, petitioner being adamant in its far-from-reasonable plea that reduced to the alternative to 'take it or leave it' labelled since Raymond Saleilles
estoppel could not be invoked by the heirs of the insured as a bar to the alleged 'contracts by adherence' (contrats d'adhesion), in contrast to those entered into by
breach of warranty and condition in the policy. lt would now rely on the fact that parties bargaining on an equal footing, such contracts (of which policies of
the insured owned a private vehicle, not a common carrier, something which it insurance and international bills of lading are prime examples) obviously call for
knew all along when not once but twice its agent, no doubt without any objection greater strictness and vigilance on the part of courts of justice with a view to
in its part, exerted the utmost pressure on the insured, a man of scant education, to protecting the weaker party from abuses and imposition, and prevent their
enter into such a contract. becoming traps for the unwary (New Civil Code. Article 24; Sent. of Supreme Court
of Spain, 13 Dec. 1934, 27 February 1942)." 8
Nor is there any merit to the second alleged error of respondent Court that no legal
liability was incurred under the policy by petitioner. Why liability under the terms of The last error assigned which would find fault with the decision of respondent Court
the policy 5 was inescapable was set forth in the decision of respondent Court of of Appeals insofar as it affirmed the lower court award for exemplary damages as
Appeals. Thus: "Since some of the conditions contained in the policy issued by the well as attorney's fees is, on its face, of no persuasive force at all.
defendant-appellant were impossible to comply with under the existing conditions
at the time and 'inconsistent with the known facts,' the insurer 'is estopped from The conclusion that inescapably emerges from the above is the correctness of the
asserting breach of such conditions.' From this jurisprudence, we find no valid decision of respondent Court of Appeals sought to be reviewed. For, to borrow
reason to deviate and consequently hold that the decision appealed from should be once again from the language of the Qua Chee Gan opinion: "The contract of
affirmed. The injured parties, to wit, Carlos Songco, Angelito Songco and Jose insurance is one of perfect good faith (uberima fides) not for the insured alone,but
Manuel, for whose hospital and medical expenses the defendant company was equally so for the insurer; in fact, it is more so for the latter, since its dominant
being made liable, were passengers of the jeepney at the time of the occurrence, bargaining position carries with it stricter responsibility." 9
and Rodolfo Songco, for whose burial expenses the defendant company was also
being made liable was the driver of the vehicle in question. Except for the fact, that This is merely to stress that while the morality of the business world is not the
they were not fare paying passengers, their status as beneficiaries under the policy morality of institutions of rectitude like the pulpit and the academe, it cannot
is recognized therein." 6 descend so low as to be another name for guile or deception. Moreover, should it
happen thus, no court of justice should allow itself to lend its approval and
Even if it be assumed that there was an ambiguity, an excerpt from the Qua Chee support.1awphîl.nèt
Gan decision would reveal anew the weakness of petitioner's contention. Thus:
"Moreover, taking into account the well known rule that ambiguities or obscurities We have no choice but to recognize the monetary responsibility of petitioner
must be strictly interpreted against the party that caused them, the 'memo of Fieldmen's Insurance Co., Inc. It did not succeed in its persistent effort to avoid
warranty' invoked by appellant bars the latter from questioning the existence of the complying with its obligation in the lower court and the Court of Appeals. Much less
appliances called for in the insured premises, since its initial expression, 'the should it find any receptivity from us for its unwarranted and unjustified plea to
undernoted appliances for the extinction of fire being kept on the premises insured escape from its liability.
hereby, ... it is hereby warranted ...,' admits of interpretation as an admission of the
existence of such appliances which appellant cannot now contradict, should the
WHEREFORE, the decision of respondent Court of Appeals of July 20, 1965, is
parol evidence rule apply." 7
affirmed in its entirety. Costs against petitioner Fieldmen's Insurance Co., Inc.

To the same effect is the following citation from the same leading case: "This rigid
application of the rule on ambiguities has become necessary in view of current
business practices. The courts cannot ignore that nowadays monopolies, cartels and
concentration of capital, endowed with overwhelming economic power, manage to
impose upon parties dealing with them cunningly prepared 'agreements' that the
47

25 SCRA 70 FIELDMEN’S INSURANCE v. MERCEDES VARGAS vda. DE SONGCO, et al. and CA


1968 / Fernando / Review of CA decision
Facts: Federico Songco, a man of scant education [first grader], owned a private jeepney.
He was induced by Fieldmen’s Insurance agent Benjamin Sambat to apply for
> In 1960, Sambat, an agent of Fieldman’s Insurance, induced Songco, a man of a Common Carrier’s Liability Insurance Policy covering his motor vehicle. [As
scant education to enter into a common carrier insurance contract with Fieldman. testified by Songco’s son Amor later,] Federico said that hisvehicle is an ‘owner’
private vehicle and not for passengers, but agent Sambat said that they can insure
> During the inducement, a son of Songco butted in and said that they could not
whatever kind of vehicle because their company is not owned by the government,
accept the type of insurance offered because theirs was an owner-type jeepney and so they could do what they please whenever they believe a vehicle is insurable.
not a common carrier. Songco paid an annual premium and he was issued a Common Carriers Accident
Insurance Policy. After the policy expired, he renewed the policy. During the
> Sambat answered that it did not matter because the insurance company was not
effectivity of the renewed policy, the insured vehicle while being driven by Rodolfo
owned by the government and therefore had nothing to do with rules and Songco [duly licensed driver and Federico’s son] collided with a car. As a result,
regulations of the latter (Fieldman). Federico and Rodolfo died, while Carlos (another son) and his wife Angelita, and a
> The insurance was executed and approved for a year from Sept. 1960-1961. It family friend sustained physical injuries.
The lower court held that Fieldmen’s Insurance cannot escape
was renewed in 1961 for another year.
liability under a common carrier insurance policy on the pretext that what was
> In Oct. 1961, the jeepney collided with a car in Bulacan and as a result, Sonco insured was a private vehicle and not a common carrier, the policy being issued
died. The remaining members of the family claimed the proceeds of the insurance upon the agent’s insistence. CA affirmed the lower court.
with the company but it refused to pay on the ground that the vehicle was not a CA DECISION AFFIRMED; FIELDMEN’S INSURANCE IS LIABLE
From Qua Chee Gan v. Law Union and Rock Insurance – Where inequitable conduct
common carrier.
is shown by an insurance firm, it is estopped from enforcing forfeitures in its
favor, in order to forestall fraud or imposition on the insured. Estoppel is primarily
Issue: based on the doctrine of good faith and the avoidance of harm that will befall the
Whether or not the Songcos’ can claim the insurance proceeds despite the fact that innocent party due to its injurious reliance.
the vehicle concerned was an owner and not a common carrier. Fieldmen’s Insurance incurred legal liability under the policy. Since some of the
conditions in the policy were impossible to comply with under the existing
conditions at the time and inconsistent with the known facts, the insurer is
Held:
estopped from asserting breach of such conditions. Except for the fact that the
Yes. passengers were not fare-paying, their status as beneficiaries under the policy is
recognized. Even if the be assumed that there was an ambiguity, such must
The company is estopped from asserting that the vehicle was not covered. After it bestrictly interpreted against the party that caused them.
had led Federico Songco to believe that he could qualify under the common carrier The contract of insurance is one of perfect good faith (uberrima fides) not for the
liability insurance policy, and to enter into a contract of insurance paying the insured alone, but equally so for the insurer; in fact, it is more so for the latter,
premiums due, it could not thereafter be permitted to change its stand to the since its dominant bargaining position carries with it stricter responsibility.
detriment of the heirs of the insured. It knew all along that Frederico owned a
FIELDMEN’S INSURANCE CO. vs. VDA. DESONGCOFACTS:
private vehicle. Its agent Sambat twice exerted the utmost pressure on the insured,
F e d e r i c o S o n g c o o w n e d a p r i v a t e j e e p n e y . O n Septem
a man of scant education, and the company did not object to this.
ber 15, 1960, he was induced by
Fieldmen'si n s u r a n c e a g e n t B e n j a m i n S a m b a t t o a p p l y f o r a
Fieldmen’s Insurance v. vda. de Songco Common Carrier's Liability Insurance Policy coveringhis motor vehicle.
He was issued a Common Carriers A c c i d e n t I n s u r a n c e P o l i c y .
48

O n t h e n e x t y e a r , h e renewed the policy by paying the annual t h a t t h e w e a k e r party may not change one whit, his participation in
premium.D u r i n g t h e e f f e c t i v i t y o f t h e r e n e w e d p o l i c y , t h e theagreement being reduced to the alternative of
insured vehicle collided with another car while beingdriven by Rodolfo “take ito r l e a v e i t ” l a b e l l e d s i n c e R a y m o n d S a l e i l l e s a s
Songco, a duly licensed driver and contracts by adherence (contrats d'adhesion), in contrast to those entered into
son of Federico (the vehicle owner). by parties bargaining onan equal footing, such contracts (i.e. insurance policies&
A s a r e s u l t , Federico Songco (father) and Rodolfo Songco (son)died, international bills of lading) obviously call for greaterstrictness and vigilance on the
along with other passengers. part of courts of justice
w i t h a v i e w t o p r o t e c t i n g t h e w e a k e r p a r t y f r o m abuses.
A c l a i m w a s f i l e d b u t w a s d e n i e d b y t h e i n s u r a n c e company on the Citing the case of Qua Chee Gan vs. Law Union & Rock I n s u r a n c e , " T h e
pretext that what was insured was aprivate vehicle and not a common carrier. c o n t r a c t o f i n s u r a n c e i s o n e o f perfect good faith (uberima fides) not
During thetrial, it was declared by a witness that when insuranceagent Benjamin for the insuredalone but equally so for the insurer; in fact, it is moreso for the latter,
Sambat was inducing Songco to insurehis vehicle, the latter butted in saying, “Our since its dominant bargaining positioncarries with it stricter responsibility."
vehicle isa p r i v a t e v e h i c l e a n d n o t f o r p a s s e n g e r s . ” B u t t h e agent
replied: “Regardless of whether your vehicle wasa n o w n e r -
type or for passengers, it could still beinsured because our co
m p a n y i s n o t o w n e d b y t h e Government. And the Government has
nothing to dowith our company.”
The Court of Appeals rendered a decision in favor of the claimants. It held
that where inequitable conduct is
s h o w n b y a n i n s u r a n c e f i r m , i t i s e s t o p p e d f r o m enforcing
forfeitures in its favor, in order to forestallfraud or imposition on the
insured. After Fieldmen's Insurance Co. had led the insured Songco to
believethat he could qualify under the common carrier liability insurance policy, it
could not, thereafter, be permitted to change its stand to the detriment of the heirs
of theinsured. The failure to apply the Doctrine of Estoppel inthis case would result in a gross
travesty of justice.
ISSUE:
Whether or not the insurance claim is proper?
RULING:
The fact that the insured owned a private vehicle, nota common carrier, was
something which the companyknew all along. In fact, it exerted the utmost
pressureon the insured, a man of scant education, to enter intothe contract of
insurance. The Court of Appeals als oheld that since some of the conditions
contained in
thep o l i c y w e r e i m p o s s i b l e t o c o m p l y w i t h u n d e r t h e existing
conditions at the time, the insurer is estoppedfrom asserting breach of such
conditions. The Supreme Court, in affirming the decision of theCour t of
Appeals, took judicial notice of the fact
thatnowadays, monopolies, cartels and concentration of capital, endowed
with overwhelming economic power,
manage to impose upon parties dealing with them
cunningly prepared agreements
49

EN BANC Dr. Gregorio Valdez, for the purpose of securing the Company's approval of the
application so that the policy to be issued thereon might be credited to said agent
G.R. No. L-47593 December 29, 1943 in connection with the inter-provincial contest which the Company was then
holding among its soliciting agents to boost the sales of its policies. Agent David
THE INSULAR LIFE ASSURANCE CO., LTD., petitioner, bribed Medical Examiner Valdez with money which the former borrowed from the
vs. applicant's mother by way of advanced payment on the premium, according to the
SERAFIN D. FELICIANO ET AL., respondents. finding of the Court of Appeals. Said court also found that before the insured signed
the application he, as well as the members of his family, told the agent and the
medical examiner that he had been sick and coughing for some time and that he
OZAETA, J.:
had gone three times to the Santol Sanatorium and had X-ray pictures of his lungs
taken; but that in spite of such information the agent and the medical examiner
In a four-to-three decision promulgated on September 13, 1941, 1 this Court
told them that the applicant was a fit subject for insurance.
affirmed the judgment of the Court of Appeals in favor of the respondents and
against the petitioner for the sum of P25,000, representing the value of two
Each of the policies sued upon contains the following stipulations:
insurance policies issued by the petitioner on the life of Evaristo Feliciano. A motion
to reconsider and set aside said decision has been filed by the petitioner, and both
parties have submitted exhaustive and luminous written arguments in support of This policy and the application herefor constitute the entire contract
their respective contentions. between the parties hereto. . . . Only the President, or the Manager, acting
jointly with the Secretary or Assistant Secretary (and then only in writing
signed by them) have power in behalf of the Company to issue permits, or
The facts of the case are set forth in the majority and dissenting opinions
to modify this or any contract, or to extend the same time for making any
heretofore handed down by this Court, the salient points of which may be briefly
premium payment, and the Company shall not be bound by any promise or
restated as follows:
representation heretofore or hereafter given by any person other than the
above-named officials, and by them only in writing and signed conjointly as
Evaristo Feliciano, who died on September 29, 1935, was suffering with advanced
stated.
pulmonary tuberculosis when he signed his applications for insurance with the
petitioner on October 12, 1934. On that same date Doctor Trepp, who had taken X-
The application contains, among others, the following statements:
ray pictures of his lungs, informed the respondent Dr. Serafin D. Feliciano, brother
of Evaristo, that the latter "was already in a very serious ad practically hopeless
condition." Nevertheless the question contained in the application — "Have you 18. — I [the applicant] hereby declare that all the above statements and
ever suffered from any ailment or disease of the lungs, pleurisy, pneumonia or answers as well as all those that I may make to the Company's Medical
asthma?" — appears to have been answered , "No" And above the signature of the Examiner in continuation of this application, to be complete, true and
applicant, following the answers to the various questions propounded to him, is the correct to the best of my knowledge and belief, and I hereby agree as
following printed statement:1awphil.net follows:

I declare on behalf of myself and of any person who shall have or claim any 1. That his declaration, with the answers to be given by me to the Medical
interest in any policy issued hereunder, that each of the above answers is Examiner, shall be the basis of the policy and form part of same.
full, complete and true, and that to the best of my knowledge and belief I
am a proper subject for life insurance. (Exhibit K.) 3. That the said policy shall not take effect until the first premium has been
paid and the policy has been delivered to and accepted by me, while I am in
The false answer above referred to, as well as the others, was written by the good health.
Company's soliciting agent Romulo M. David, in collusion with the medical examiner
50

4. That the agent taking this application has no authority to make, modify We cannot bring ourselves to believe that the insured did not take the trouble to
or discharge contracts, or to waive any of the Company's rights or read the answers contained in the photostatic copy of the application attached to
requirements. and made a part of the policy before he accepted it and paid the premium thereon.
He must have notice that the answers to the questions therein asked concerning his
5. My acceptance of any policy issued on this application will constitute a clinical history were false, and yet he accepted the first policy and applied for
ratification by me of any corrections in or additions to this application another. In any event, he obligated himself to read the policy when he subscribed
made by the Company in the space provided "For Home Office Corrections to this statement: "My acceptance of any policy issued on this application will
or Additions Only." I agree that photographic copy of this applications as constitute a ratification by me of any corrections in or additions to this application
corrected or added to shall constitute sufficient notice to me of the made by the Company . . ." By accepting the policy he became charged with
changes made. (Emphasis added.) knowledge of its contents, whether he actually read it or not. He could not ostrich-
like hide his head from it in order to avoid his part of the bargain and at the same
The petitioner insists that upon the facts of the case the policies in question are null time claim the benefit thereof. He knew, or was chargeable with knowledge, from
and void ab initio and that all that the respondents are entitled to is the refund of the very terms of the two policies sued upon (one of which is printed in English and
the premiums paid thereon. After a careful re-examination of the facts and the law, the other in Spanish) that the soliciting agent and the medical examiner had no
we are persuaded that petitioner's contention is correct. To the reasons adduced in power to bind the Company by any verbal promise or oral representation. The
the dissenting opinion heretofore published, we only desire to add the following insured, therefore, had no right to rely — and we cannot believe he relied in good
considerations: faith — upon the oral representation. The insured, therefore, had no right to rely —
and we cannot believe he relied in good faith — upon the oral representation of
said agent and medical examiner that he (the applicant) was a fit subject for
When Evaristo Feliciano, the applicant for insurance, signed the application in blank
insurance notwithstanding that he had been and was still suffering with advanced
and authorized the soliciting agent and/or medical examiner of the Company to
pulmonary tuberculosis.
write the answers for him, he made them his own agents for that purpose, and he
was responsible for their acts in that connection. If they falsified the answers for
him, he could not evade the responsibility for he falsification. He was not supposed From all the facts and circumstances of this case, we are constrained to conclude
to sign the application in blank. He knew that the answers to the questions therein that the insured was a coparticipant, and coresponsible with Agent David and
contained would be "the basis of the policy," and for that every reason he was Medical Examiner Valdez, in the fraudulent procurement of the policies in question
required with his signature to vouch for truth thereof. and that by reason thereof said policies are void ab initio.

Moreover, from the facts of the case we cannot escape the conclusion that the Wheretofore, the motion for reconsideration is sustained and the judgment of the
insured acted in connivance with the soliciting agent and the medical examiner of Court of Appeals is hereby reversed. Let another judgment be entered in favor of
the Company in accepting the policies in question. Above the signature of the the respondents and against the petitioner for the refund of the premiums
applicant is the printed statement or representation: " . . . I am a proper subject for amounting to P1,389, with legal interest thereon from the date of the complaint,
life insurance." In another sheet of the same application and above another and without any finding as to costs.
signature of the applicant was also printed this statement: "That the said policy
shall not take effect until he first premium has been paid and the policy as been
delivered to and accepted by me, while I am in good health." When the applicant
signed the application he was "having difficulty in breathing, . . . with a very high
fever." He had gone three times to the Santol Sanatorium and had X-ray pictures
taken of his lungs. He therefore knew that he was not "a proper subject for life
insurance." When he accepted the policy, he knew that he was not in good health.
Nevertheless, he not only accepted the first policy of P20,000 but then and there
applied for and later accepted another policy of P5,000.
51

74 Phil. 468 – Mercantile Law – Insurance Law – Representation – Insurance Agent’s


Fraud

From the court’s decision rendered in the case of Insular Life Assurance vs
Feliciano(1941), Insular Life filed a motion for reconsideration. Insular avers that
Feliciano is not entitled to the claim because the insurance policy is void ab initio;
that he connived with the insurance agent and the medical examiner; and that at
best, Feliciano is only entitled to refund or the reimbursement of what he has paid
in premium.

ISSUE: Whether or not Insular Life is correct.

HELD: Yes. This time, the Supreme Court held that Insular Life’s contention is
correct. When Evaristo Feliciano, the applicant for insurance, signed the application
in blank and authorized the soliciting agent and/or medical examiner of Insular to
write the answers for him, he made them his own agents for that purpose, and he
was responsible for their acts in that connection. If they falsified the answers for
him, he could not evade the responsibility for the falsification. He was not supposed
to sign the application in blank. He knew that the answers to the questions therein
contained would be “the basis of the policy,” and for that very reason he was
required with his signature to vouch for truth thereof.
52

EN BANC examiner that the applicant had been sick and coughing for some time and that he
had also gone three times to the Santol Sanitarium. On appeal, this finding of facts
G.R. No. L-47593 September 13, 1941
of the lower court was sustained by the Court of Appeals. This concludes the
THE INSULAR LIFE ASSURANCE CO., LTD., petitioner, controversy over the facts in so far as this Court is concerned.
vs. The first assignment of error of the petitioner raises the question we are now called
SERAFIN D. FELICIANO and ANGEL, FLORENDA, EUGENIO, HERMINIO and LETICIA, upon to decide:
all surnamed FELICIANO, represented by their guardian ad litem SERAFIN D. The Court of Appeals erred in holding that an insurance company has no right to
FELICIANO, respondents. avoid a policy where its agent knowingly and intentionally wrote down the answers
Araneta, Zaragosa, Araneta & Bautista for petitioner. in the application differing from those made by the insured, in disregard of the
Delfin Joven for respondents. exception that when the agent, instead of serving the interests of his principal, acts
in his own or another’s interest and adversely to that of his principal, the said
principal is not bound by said acts of the agent.”
DECISION
On the proposition thus presented, there are two main avenues of approach
LAUREL, J.: indicated: one leading to the validation of a policy where its agent, without fraud,
One Evaristo Feliciano filed an application for insurance with the herein petitioner collusion or bad faith on the part of the insured, falsified the answers given by the
upon the solicitation of one of its agents. Two insurance policies to the aggregate insured; and the other, leading to the avoidance of the policy under the
amount of P25,000 were issued to him. Feliciano died on September 29, 1935. The circumstances. We see no need for an extended discussion of the conflicting
defendant company refused to pay on the ground that the policies were authorities. Whenever courts are given the choice between two conflicting
fraudulently obtained, the insured having given false answers and statements in the principles, the determinative fact which should sway them is the conformity of its
application as well as in the medical report. The present action was brought to contemplated course to reason and to “the common sense of the situation.” The
recover on said policies. The lower court rendered judgment in favor of the life of the law is not only logic but experience.
plaintiffs. The lower court found that at the time Feliciano filed his application and The phenomenal growth of insurance from almost nothing a hundred years ago to
at the time he was subjected to physical examination by the medical examiner of its present gigantic proportion is not of the outstanding marvels of present-day
the herein petitioner, he was already suffering from tuberculosis. This fact appears business life. The demand for economic security, the growing need for social
in the negative both in the application and in the medical report. The lower court, stability, and the clamor for protection against the hazards of cruel-crippling
after an exhaustive examination of the conflicting testimonies, also found that calamities and sudden economic shocks, have made insurance one of the felt
Feliciano was made to sign the application and the examiner’s report in blank, and necessities of modern life. Insurance is no longer a rich man’s monopoly. Upon it
that afterwards the blank spaces therein were filled in by the agent and the medical are heaped the assured hopes of many families of modest means. It is woven, as it
examiner, who made it appear therein that Feliciano was a fit subject for insurance. were, into the very warp and woof of national economy. It touches the holiest and
The lower court also held that neither the insured nor any member of his family most sacred ties in the life of man-love of parents, love of wives and love of
concealed the real state of health of the insured. That as a matter of fact the children. It is of common knowledge that the selling of insurance today is subjected
insured, as well as the members of his family, told the agent and the medical to the whirlwind pressure of modern salesmanship. Insurance companies send
53

detailed instructions to their agents to solicit and procure applications. These regard to the subject matter of the answers or the nature of the agent’s duties or
agents are to be found all over the length and breadth of the land. They are limitations on his authority, at least if not brought to the attention of the applicant.
stimulated to more active efforts by contests and by the keen competition offered
The fact that the insured did not read the application which he signed, is not
by other rival insurance companies. They are supplied with blank applications and
indicative of bad faith. It has been held that it is not negligence for the insured to
paid large commissions on the policies secured by them. All transactions are
sign an application without first reading it if the insurer by its conduct in appointing
generally done through these agents. They act, in fact and in theory, as the general
the agent influenced the insured to place trust and confidence in the agent. (Den
representatives of the insurance companies. They supply all the information ,
Hartog v. Home Nat. Ins. Asso., 197 Iowa, 143 196 N. W. 944.) As the court said in
prepare and answer the applications, submit the applications to their companies,
the case of Germania L. Ins. Co. v. Lunkebiemer, 127 Ind. 538, 26 N. E. 1082, “Nor
conclude the transactions, and otherwise smooth out all difficulties. The agents, in
can it be said that the assured, who has fully, frankly, truthfully, and in good faith
short, do what the company set them to do.
answered all the required questions, is guilty of negligence in signing, without
In the present case, the agent knew all the time the true state of health of the reading, the application which is thereupon prepared by the agent. He is justified in
insured. The insurer’s medical examiner approve the application knowing full well assuming that the agent has, with equal good faith, truthfully recorded the answers
that the applicant was sick. The situation is one in which one of two innocent given him. He may well say to the company: “You accredited this man to me as your
parties must bear a loss for his reliance upon a third person. In this case, it was the representative and I signed the application thus prepared by him, relying upon the
insurer who gave the agent authority to deal with the applicant. It was the one who character which you gave him when you commissioned him to come to me as your
selected the agent, thus implying that the insured could put his trust on him. It was agent. If he acted dishonestly in the matter, you and not I must suffer the
the one who drafted and accepted the policy and consummated the contract. It consequences.’ …” In the instant case, it has been proved that the insured could not
seems reasonable that as between the two of them, the one who employed and read English, the language in which the application was written, and that after the
gave character to the third person as its agent should be the one to bear the loss. contract was signed, it was kept by his mother. As a consequence, the insured had
no opportunity to read or correct any misstatement therein. (Bill of Exceptions, pp.
The company received the money of the applicant as the price of the risk to be
60-61.)
taken by it. If the policy should be avoided, it must be because it was void from the
very beginning, and the result would be that the insurer, while it received the We have not been insensible to the appeal that the course we have followed may
money, never assumed any risk. The result would be, in the language of one of the lead to fraud and work hardship on insurance companies, for it would be easy for
cases, “to place every simple or uneducated person seeking insurance at the mercy insurance agents and applicants to insert false answers in their applicants to insert
of the insurer who could, through its agent, insert in every application, unknown to false answers in their applications for insurance. This means that it is to the
the applicant and over his signature, some false statements which would enable particular interest of these companies to exercise greater care in the selection of
him to avoid all liability while retaining the price paid for the supposes insurance.” their agents and examiners. Their protection is still in their own hands and which
(State Insurance Company v. Taylor, 14 Colo. 499, 24 Pac. 333.) The weight of may be achieved by other means. Withal, the attainment of a common good may
authority is that if an agent of the insurer, after obtaining from an applicant for involve impairment and even sacrifice of beneficial interests of a particular group,
insurance a correct and truthful answer to interrogatories contained in the but in life, compromise is inevitable until the hour of doom strikes.
application for insurance, without knowledge of the applicant fills in false answers,
The petition is hereby DISMISSED and the judgment sought to be reviewed
either fraudulently or otherwise, the insurer cannot assert the falsity of such
is AFFIRMED with costs against the petitioner. SO ORDERED.
answers as a defense to liability on the policy, and this is true generally without
54

3 Phil. 201 (40 Off. Gaz. 2842) – Mercantile Law – Insurance Law – Representation > Evaristo Feliciano filed an application with Insular Life upon the solicitation of one
– Insurance Agent’s Fraud of its agents.

Evaristo Feliciano was issued an insurance policy by Insular Life. In September 1935, > It appears that during that time, Evaristo was already suffering from
he died. His heirs (Serafin Feliciano et al) filed an insurance claim but Insular Life tuberculosis. Such fact appeared during the medical exam, but the examiner and
the company’s agent ignored it.
denied the application as it averred that Feliciano’s application was attended by
fraud. It was later found in court that the insurance agent and the medical examiner > After that, Evaristo was made to sign an application form and thereafter the
of Insular Life who assisted Feliciano in signing the application knew that Feliciano blank spaces were filled by the medical examiner and the agent making it appear
was already suffering from tuberculosis; that they were aware of the true medical that Evaristo was a fit subject of insurance. (Evaristo could not read and understand
condition of Feliciano yet they still made it appear that he was healthy in the English)
insurance application form; that Feliciano signed the application in blank and the > When Evaristo died, Insular life refused to pay the proceeds because of
agent filled the information for him. concealment.

ISSUE: Whether or not Insular Life can avoid the insurance policy by reason of the Issue:
fact that its agent knowingly and intentionally wrote down the answers in the Whether or not Insular Life was bound by their agent’s acts.
application differing from those made by Feliciano hence instead of serving the
interests of his principal, acts in his own or another’s interest and adversely to that Held:
of his principal. Yes.
HELD: No. Insular Life must pay the insurance policy. The weight of authority is that The insurance business has grown so vast and lucrative within the past century.
if an agent of the insurer, after obtaining from an applicant for insurance a correct Nowadays, even people of modest means enter into insurance contracts. Agents
and truthful answer to interrogatories contained in the application for insurance, who solicit contracts are paid large commissions on the policies secured by them.
without knowledge of the applicant fills in false answers, either fraudulently or They act as general representatives of insurance companies.
otherwise, the insurer cannot assert the falsity of such answers as a defense to IN the case at bar, the true state of health of the insured was concealed by the
liability on the policy, and this is true generally without regard to the subject matter agents of the insurer. The insurer’s medical examiner approved the application
of the answers or the nature of the agent’s duties or limitations on his authority, at knowing fully well that the applicant was sick. The situation is one in which of two
least if not brought to the attention of the applicant. innocent parties must bear a loss for his reliance upon a third person. In this case, it
is the one who drafted and accepted the policy and consummated the contract. It
The fact that the insured did not read the application which he signed, is not seems reasonable that as between the two of them, the one who employed and
indicative of bad faith. It has been held that it is not negligence for the insured to gave character to the third person as its agent should be the one to bear the loss.
sign an application without first reading it if the insurer by its conduct in appointing Hence, Insular is liable to the beneficiaries.
the agent influenced the insured to place trust and confidence in the agent.

73 PHIL 201

Facts:
55

SECOND DIVISION This opinion of the Insurance Commissioner notwithstanding, appellant refused to
settle its obligation.
G.R. No. L-30685 May 30, 1983
Appellant alleged that the insured was guilty of misrepresentation when he
NG GAN ZEE, plaintiff-appellee, answered "No" to the following question appearing in the application for life
vs. insurance-
ASIAN CRUSADER LIFE ASSURANCE CORPORATION, defendant-appellant.
Has any life insurance company ever refused your application for insurance
ESCOLIN, J.: or for reinstatement of a lapsed policy or offered you a policy different
from that applied for? If, so, name company and date.
This is an appeal from the judgment of the Court of First Instance of Manila,
ordering the appellant Asian-Crusader Life Assurance Corporation to pay the face In its brief, appellant rationalized its thesis thus:
value of an insurance policy issued on the life of Kwong Nam the deceased husband
of appellee Ng Gan Zee. Misrepresentation and concealment of material facts in ... As pointed out in the foregoing summary of the essential facts in this
obtaining the policy were pleaded to avoid the policy. The lower court rejected the case, the insured had in January, 1962, applied for reinstatement of his
appellant's theory and ordered the latter to pay appellee "the amount of P lapsed life insurance policy with the Insular Life Insurance Co., Ltd, but this
20,000.00, with interest at the legal rate from July 24, 1964, the date of the filing of was declined by the insurance company, although later on approved for
the complaint, until paid, and the costs. " reinstatement with a very high premium as a result of his medical
examination. Thus notwithstanding the said insured answered 'No' to the
The Court of Appeals certified this appeal to Us, as the same involves solely a [above] question propounded to him. ... 1
question of law.
The lower court found the argument bereft of factual basis; and We quote with
On May 12, 1962, Kwong Nam applied for a 20-year endowment insurance on his approval its disquisition on the matter-
life for the sum of P20,000.00, with his wife, appellee Ng Gan Zee as beneficiary. On
the same date, appellant, upon receipt of the required premium from the insured, On the first question there is no evidence that the Insular Life Assurance
approved the application and issued the corresponding policy. On December 6, Co., Ltd. ever refused any application of Kwong Nam for insurance. Neither
1963, Kwong Nam died of cancer of the liver with metastasis. All premiums had is there any evidence that any other insurance company has refused any
been religiously paid at the time of his death. application of Kwong Nam for insurance.

On January 10, 1964, his widow Ng Gan Zee presented a claim in due form to ... The evidence shows that the Insular Life Assurance Co., Ltd. approved
appellant for payment of the face value of the policy. On the same date, she Kwong Nam's request for reinstatement and amendment of his lapsed
submitted the required proof of death of the insured. Appellant denied the claim on insurance policy on April 24, 1962 [Exh. L-2 Stipulation of Facts, Sept. 22,
the ground that the answers given by the insured to the questions appealing in his 1965). The Court notes from said application for reinstatement and
application for life insurance were untrue. amendment, Exh. 'L', that the amount applied for was P20,000.00 only and
not for P50,000.00 as it was in the lapsed policy. The amount of the
Appellee brought the matter to the attention of the Insurance Commissioner, the reinstated and amended policy was also for P20,000.00. It results,
Hon. Francisco Y. Mandamus, and the latter, after conducting an investigation, therefore, that when on May 12, 1962 Kwong Nam answered 'No' to the
wrote the appellant that he had found no material concealment on the part of the question whether any life insurance company ever refused his application
insured and that, therefore, appellee should be paid the full face value of the policy. for reinstatement of a lapsed policy he did not misrepresent any fact.
56

... the evidence shows that the application of Kwong Nam with the Insular The question to be resolved may be propounded thus: Was appellant, because of
Life Assurance Co., Ltd. was for the reinstatement and amendment of his insured's aforesaid representation, misled or deceived into entering the contract or
lapsed insurance policy-Policy No. 369531 -not an application for a 'new in accepting the risk at the rate of premium agreed upon?
insurance policy. The Insular Life Assurance Co., Ltd. approved the said
application on April 24, 1962. Policy No. 369531 was reinstated for the The lower court answered this question in the negative, and We agree.
amount of P20,000.00 as applied for by Kwong Nam [Exhs. 'L', 'L-l' and 'L-
2']. No new policy was issued by the Insular Life Assurance Co., Ltd. to Section 27 of the Insurance Law [Act 2427] provides:
Kwong Nam in connection with said application for reinstatement and
amendment. Such being the case, the Court finds that there is no
Sec. 27. Such party a contract of insurance must communicate to the
misrepresentation on this matter. 2
other, in good faith, all facts within his knowledge which are material to
the contract, and which the other has not the means of ascertaining, and
Appellant further maintains that when the insured was examined in connection as to which he makes no warranty. 3
with his application for life insurance, he gave the appellant's medical examiner
false and misleading information as to his ailment and previous operation. The
Thus, "concealment exists where the assured had knowledge of a fact material to
alleged false statements given by Kwong Nam are as follows:
the risk, and honesty, good faith, and fair dealing requires that he should
communicate it to the assurer, but he designedly and intentionally withholds the
Operated on for a Tumor [mayoma] of the stomach. Claims that Tumor has same." 4
been associated with ulcer of stomach. Tumor taken out was hard and of a
hen's egg size. Operation was two [2] years ago in Chinese General
It has also been held "that the concealment must, in the absence of inquiries, be
Hospital by Dr. Yap. Now, claims he is completely recovered.
not only material, but fraudulent, or the fact must have been intentionally
withheld." 5
To demonstrate the insured's misrepresentation, appellant directs Our attention to:
Assuming that the aforesaid answer given by the insured is false, as claimed by the
[1] The report of Dr. Fu Sun Yuan the physician who treated Kwong Nam at the appellant. Sec. 27 of the Insurance Law, above-quoted, nevertheless requires that
Chinese General Hospital on May 22, 1960, i.e., about 2 years before he applied for fraudulent intent on the part of the insured be established to entitle the insurer to
an insurance policy on May 12, 1962. According to said report, Dr. Fu Sun Yuan had rescind the contract. And as correctly observed by the lower court,
diagnosed the patient's ailment as 'peptic ulcer' for which, an operation, known as a "misrepresentation as a defense of the insurer to avoid liability is an 'affirmative'
'sub-total gastric resection was performed on the patient by Dr. Pacifico Yap; and defense. The duty to establish such a defense by satisfactory and convincing
evidence rests upon the defendant. The evidence before the Court does not clearly
[2] The Surgical Pathology Report of Dr. Elias Pantangco showing that the specimen and satisfactorily establish that defense."
removed from the patient's body was 'a portion of the stomach measuring 12 cm.
and 19 cm. along the lesser curvature with a diameter of 15 cm. along the greatest It bears emphasis that Kwong Nam had informed the appellant's medical examiner
dimension. that the tumor for which he was operated on was "associated with ulcer of the
stomach." In the absence of evidence that the insured had sufficient medical
On the bases of the above undisputed medical data showing that the insured was knowledge as to enable him to distinguish between "peptic ulcer" and "a tumor",
operated on for peptic ulcer", involving the excision of a portion of the stomach, his statement that said tumor was "associated with ulcer of the stomach, " should
appellant argues that the insured's statement in his application that a tumor, "hard be construed as an expression made in good faith of his belief as to the nature of
and of a hen's egg size," was removed during said operation, constituted material his ailment and operation. Indeed, such statement must be presumed to have been
concealment. made by him without knowledge of its incorrectness and without any deliberate
intent on his part to mislead the appellant.
57

While it may be conceded that, from the viewpoint of a medical expert, the
Has any life insurance company ever refused your application for insurance or for
information communicated was imperfect, the same was nevertheless sufficient to
have induced appellant to make further inquiries about the ailment and operation reinstatement of a lapsed policy or offered you a policy different from that applied
of the insured.
for? If, so, name company and date.
Section 32 of Insurance Law [Act No. 24271 provides as follows:
Kwong Nam answered “No” to the above question.

Section 32. The right to information of material facts maybe waived either Kwong Nam was also examined by Asian Crusader’s medical examiner to whom he
by the terms of insurance or by neglect to make inquiries as to such facts
where they are distinctly implied in other facts of which information is disclosed that he was once operated and a tumor was removed from his stomach
communicated. and such was “associated with ulcer of the stomach.”

It has been held that where, upon the face of the application, a question appears to Kwong Nam’s application was approved. In May 1963, he died. His widow, Ng Gan
be not answered at all or to be imperfectly answered, and the insurers issue a
policy without any further inquiry, they waive the imperfection of the answer and Zee, filed an insurance claim but Asian Crusader refused her claim as it insisted that
render the omission to answer more fully immaterial. 6 Kwong Nam concealed material facts from them when he was applying for the

As aptly noted by the lower court, "if the ailment and operation of Kwong Nam had insurance; that he misrepresented the fact that he was actually denied application
such an important bearing on the question of whether the defendant would by Insular Life when he was renewing his application with them; that Kwong Nam
undertake the insurance or not, the court cannot understand why the defendant or
its medical examiner did not make any further inquiries on such matters from the was actually operated for peptic ulcer.
Chinese General Hospital or require copies of the hospital records from the
appellant before acting on the application for insurance. The fact of the matter is ISSUE: Whether or not Ng Gan Zee can collect the insurance claim.
that the defendant was too eager to accept the application and receive the
insured's premium. It would be inequitable now to allow the defendant to avoid HELD: Yes. Asian Crusader was not able to prove that Kwong Nam’s statement that
liability under the circumstances." Insular Life did not deny his insurance renewal with them is untrue. In fact,

Finding no reversible error committed by the trial court, the judgment appealed evidence showed that in April 1962, Insular Life approved Kwong Nam’s request of
from is hereby affirmed, with costs against appellant Asian-Crusader life Assurance reinstatement only with the condition that Kwong Nam’s plan will be lowered from
Corporation.
P50,000.00 to P20,000.00 considering his medical history.
SO ORDERED.
Kwong Nam did not conceal anything from Asian Crusader. His statement that his
operation, in which a tumor the size of a hen’s egg was removed from his stomach,
122 SCRA 461 – Mercantile Law – Insurance Law – Concealment –
was only “associated with ulcer of the stomach” and not peptic ulcer can be
Misrepresentation – Duty of Insurance Company to Make Inquiry
considered as an expression made in good faith of his belief as to the nature of his
In May 1962, Kwong Nam applied for a 20-year endowment policy with Asian
ailment and operation. Indeed, such statement must be presumed to have been
Crusader Life Assurance Corporation. Asian Crusader asked the following question:
58

made by him without knowledge of its incorrectness and without any deliberate Has any life insurance company ever refused your application for insurance or for

intent on his part to mislead Asian Crusader. reinstatement of a lapsed policy or offered you a policy different from that applied
for? If, so, name company and date.
While it may be conceded that, from the viewpoint of a medical expert, the The lower court ruled against the company on lack of evidence.
information communicated was imperfect, the same was nevertheless sufficient to Appellant further maintains that when the insured was examined in connection
have induced Asian Crusader to make further inquiries about the ailment and with his application for life insurance, he gave the appellant's medical examiner
operation of Kwong Nam. It has been held that where, upon the face of the false and misleading information as to his ailment and previous operation. The
company contended that he was operated on for peptic ulcer 2 years before the
application, a question appears to be not answered at all or to be imperfectly
policy was applied for and that he never disclosed such an operation.
answered, and the insurers issue a policy without any further inquiry, they waive
the imperfection of the answer and render the omission to answer more fully
Issue: WON Asian Crusader was deceived into entering the contract or in accepting
immaterial.
the risk at the rate of premium agreed upon because of insured's representation?
Facts:
Kwong Nam applied for a 20-year endowment insurance on his life for the sum of Held: No. Petition dismissed.
P20,000.00, with his wife, appellee Ng Gan Zee as beneficiary. On the same date,
Asian Crusader, upon receipt of the required premium from the insured, approved Ratio:
the application and issued the corresponding policy. Kwong Nam died of cancer of Section 27 of the Insurance Law:
the liver with metastasis. All premiums had been paid at the time of his death. Sec. 27. Such party a contract of insurance must communicate to the other, in good
Ng Gan Zee presented a claim for payment of the face value of the policy. On the faith, all facts within his knowledge which are material to the contract, and which
same date, she submitted the required proof of death of the insured. Appellant the other has not the means of ascertaining, and as to which he makes no warranty.
denied the claim on the ground that the answers given by the insured to the "Concealment exists where the assured had knowledge of a fact material to the
questions in his application for life insurance were untrue. risk, and honesty, good faith, and fair dealing requires that he should communicate
Appellee brought the matter to the attention of the Insurance Commissioner. The it to the assurer, but he designedly and intentionally withholds the same."
latter, after conducting an investigation, wrote the appellant that he had found no It has also been held "that the concealment must, in the absence of inquiries, be
material concealment on the part of the insured and that, therefore, appellee not only material, but fraudulent, or the fact must have been intentionally
should be paid the full face value of the policy. The company refused to settle its withheld."
obligation. Fraudulent intent on the part of the insured must be established to entitle the
Appellant alleged that the insured was guilty of misrepresentation when he insurer to rescind the contract. And as correctly observed by the lower court,
answered "No" to the following question appearing in the application for life "misrepresentation as a defense of the insurer to avoid liability is an 'affirmative'
insurance- defense. The duty to establish such a defense by satisfactory and convincing
59

evidence rests upon the defendant. The evidence before the Court does not clearly > In 1963, Kwong died of cancer of the liver with metastasis. Asian refused to pay
and satisfactorily establish that defense." on the ground of alse information.
It bears emphasis that Kwong Nam had informed the appellant's medical examiner
> It was found that prior to his application, Kwong was diagnosed to have peptic
of the tumor. His statement that said tumor was "associated with ulcer of the
ulcers, and that during the operation what was removed from Kwong’s body was
stomach" should be construed as an expression made in good faith of his belief as
actually a portion of the stomach and not tumor.
to the nature of his ailment and operation.
While the information communicated was imperfect, the same was sufficient to
Issue:
have induced appellant to make further inquiries about the ailment and operation
of the insured. Whether or not the contract may be rescinded on the ground of the imperfection in
Section 32 of Insurance Law: the application form.
Section 32. The right to information of material facts maybe waived either by the
terms of insurance or by neglect to make inquiries as to such facts where they are Held:

distinctly implied in other facts of which information is communicated. NO.


Where a question appears to be not answered at all or to be imperfectly answered,
and the insurers issue a policy without any further inquiry, they waive the Kwong did not have sufficient knowledge as to distinguish between a tumor and a

imperfection of the answer and render the omission to answer more fully peptic ulcer. His statement therefore was made in good faith. Asian should have

immaterial. made an inquiry as to the illness and operation of Kwong when it appeared on the
The company or its medical examiner did not make any further inquiries on such face of the application that a question appeared to be imperfectly answered.
matters from the hospital before acting on the application for insurance. The fact of Asian’s failure to inquire constituted a waiver of the imperfection in the answer.
the matter is that the defendant was too eager to accept the application and
receive the insured's premium. It would be inequitable now to allow the defendant
to avoid liability under the circumstances.

Facts:

> In 1962, Kwon Nam applied for a 20yr endowment insurance on his life with his
wife, Ng Gan Zee as the beneficiary.

> He stated in his application that he was operated on for tumor of the stomach
associated with ulcer.

Das könnte Ihnen auch gefallen